Re: [obm-l] dúvida combinatória

2002-12-06 Por tôpico Augusto César Morgado
Chamemos as pessoas de A, B, C.
Supondo os 5 livros diferentes, ha 3^5 = 243 modos de distribui-los (o 
primeiro livro pode ser distribuído de 3 modos, o segundo de 3 modos etc).
Ha  2^5 = 32 modos de distribui-los apenas a A e B, 32 a A e C etc.
Ha 1 modo de distribui-los apens a A, 1 a B etc.
A resposta eh  243 - 32 - 32 - 32 + 1 + 1 +1 = 150

rafaelc.l wrote:

Não consegui entender esta questão, gostaria de ajuda.


(UFSM-2002) De quantas maneiras podemos distribuir 5 
livros entre 3 pessoas de modo que cada pessoa receba 
pelo menos um livro?



   Obrigado


__
Venha para a VilaBOL!
O melhor lugar para você construir seu site. Fácil e grátis!
http://vila.bol.com.br


=
Instruções para entrar na lista, sair da lista e usar a lista em
http://www.mat.puc-rio.br/~nicolau/olimp/obm-l.html
O administrador desta lista é [EMAIL PROTECTED]
=




=
Instruções para entrar na lista, sair da lista e usar a lista em
http://www.mat.puc-rio.br/~nicolau/olimp/obm-l.html
O administrador desta lista é [EMAIL PROTECTED]
=



Re: [obm-l] probabilidade

2002-12-05 Por tôpico Augusto César Morgado



Ha 2^4 = 16 casos possiveis (cada elemento pode ser par ou impar, e supomos
que isso ocorra com prob. iguais).
Para ser impar, o produto da diagonal principal deve ser par e o da secundaria
impar, ou vice-versa. Para o da principal ser par, ha 3 casos (PP, PI, IP)
e para o da secundaria ser impar, 1 caso (I, I). Mais 3 casos do vice-versa,
ha 6 casos favoraveis e a resposta eh 6/16 = 3/8.
Em suma, exatamente o que voce fez (a unica finalidade desta mensagem eh
tentar explicar um pouco mais o FUI 
COM BINANDO).
Morgado 

Marcos Reynaldo wrote:

  Para o determinante ser impar, o produto dos elementosde uma das diagonais deve ser par e na outra serimpar.Eu fiz pegando todos os casos. Escrevi numa unicalinha pra ficar mais facil (a11a22,a12a21)e fuicombinando. Pelas minhas contas deu 6/16=3/8. Gostariade saber do pessoal se tem outra maneira.[]'s Marcos --- [EMAIL PROTECTED] escreveu:  Qual a probabilidadedo determinante de uma matriz
  
quadrada 2x2, com coeficientes inteiros, ser mpar? 

___Busca Yahoo!O melhor lugar para encontrar tudo o que voc procura na Internethttp://br.busca.yahoo.com/=Instrues para entrar na lista, sair da lista e usar a lista emhttp://www.mat.puc-rio.br/~nicolau/olimp/obm-l.htmlO administrador desta lista  [EMAIL PROTECTED]=






Re: [obm-l] moedas

2002-12-05 Por tôpico Augusto César Morgado



Gente, vamos ler com mais atenao a clara e correta explicaao do Marcos
Reinaldo. Voces podem usar o espao amostral 
{,kkkc,kkcc,kccc,}, mas isso eh inconveniente porque ele nao eh equiprovavel.
Quem acha que eh deve estar disposto a apostar em  contra duas caras
e duas coroas. Topa?
Morgado
Andre Linhares wrote:

  
  
  
   Acho que  a mesma coisa tirar kkkc e kckk. No iporta a ordem.
Afinal, que ordem, se elas esto sendo jogadas para ar todas de uma vez?E
se, por exemplo, asmoedas ficarem na seguinte posio, o que voc iria considerar?
kcck, cckk, kkcc ou ckkc? Acho que no h diferena nenhuma entre essas possibilidades
  
  
  kc
  
  kc
   
   Assim o espao amostral seria {,kkkc,kkcc,kccc,} e haveria
probabilidade de 1/5 = 20%.
  
  
  
From: Marcos Reynaldo Reply-To:
[EMAIL PROTECTED]  To: [EMAIL PROTECTED]  Subject: Re: [obm-l]
moedas  Date: Thu, 5 Dec 2002 02:27:28 -0300 (ART)Sim, realmente
eh a mesma coisa, mas soh tem um  detalhe, ao considerarmos isto, veja
que os resultados  no ocorrem com a mesma frequencia. Veja:  (a)
  1 vez  (b) kkkc, kkck, kckk, ckkk
.. 4 vezes  (c) kkcc, kckc, kcck, ckkc, ckck, cckk .. 6 vezes
 (d) kccc, ckcc, cckc, ccck .. 4 vezes  (e)  
1 vez  Assim o espaco amostral nao eh equiprovavel.  Ento uma outra
maneira de resolver o problema   considerar prob de (a) ocorrer (=p(a))como
referencia  e a frequencia de cada um como pesos.  Lembrando que
a soma das probabilidades dos cinco  resultados  1, temos:  p(a)+p(b)+p(c)+p(d)+p(e)=1
 -- p(a)+4p(a)+6p(a)+4p(a)+p(a)=1  donde conclui-se que p(a)=1/16
e portanto  p(c)=6p(a)=6/16=3/8.  Se vc observar eh mais facil considerar
cada ordenacao  como resultado diferente e assim todas com a mesma  probabilidade
de ocorrer.[]'s Marcos   --- pichurin 
escreveu:   Jogar as moedas e obter kkkc ou kkck no
 obter amesma coisa?Se for a mesma coisa este espao
amostral pode serreduzido, passando de {,kkkc,kkck,KKCC,kckk,KCKC,KCCK,kccc,ckkk,CKKC,CKCK,ckcc,CCKK,cckc,ccck,}
   para{,kkkc,kkcc,kccc,}  
Desculpem, mas  que no estou entendendo muito bemeste problema.
 --- pichurin 
escreveu: Mas ao jogar as moedas, obter kkkc
ou kkck no  a mesma coisa?___
 Busca Yahoo!  O melhor lugar para encontrar tudo o que voc procura
na Internet  http://br.busca.yahoo.com/  =
 Instrues para entrar na lista, sair da lista e usar a lista em  http://www.mat.puc-rio.br/~nicolau/olimp/obm-l.html
 O administrador desta lista  
=
 
  
  MSN Messenger: converse com os seus amigos online. 
Instale grtis. Clique aqui.
 = 
Instrues para entrar na lista, sair da lista e usar a lista em http://www.mat.puc-rio.br/~nicolau/olimp/obm-l.html 
O administrador desta lista   = 
  
  
  
  


Re: [obm-l] Amigo secreto...

2002-12-05 Por tôpico Augusto César Morgado



Na empresa em que trabalho, o homem dos computadores fez programa que gerou
a permutaao e expediu e-mails para cada um comunicando quem era o amigo
oculto, sem que ninguem visse a permutaao.
Morgado

Eduardo Azevedo wrote:

   verdade que o jeito comum, s tem e^-1 de chance de nao "dar certo", masai e so tirar outro papelzinho.A pior coisa desse mtodo so os ciclos pequenos (que quase sempreacontecem).Por outro lado, se fizer a permutao, a principio, ninguem sabe pra quemvai dar presente. E isso  um problema bem maior, j que voc no sabe secompra perfume de homem ou de mulher, CD de forr ou de rock.Pra contornar isso, o sorteio teria que ser um pouquinho mais complicado doque no mtodo usual.- Original Message -From: "Augusto Csar Morgado" [EMAIL PROTECTED]To: [EMAIL PROTECTED]Sent: Wednesday, December 04, 2002 7:08 PMSubject: Re: [obm-l] A
migo secreto...
  
Um processo extremamente eficiente de fazer um sorteio de amigo ocultoeh fazer uma permutaao (isto eh, colocar os nomes das pessoas em fila)das pessoas. Ai o primeiro da fila presenteia o segundo, o segundopresenteia o terceiro,..., o ultimo presenteia o primeiro. Tal processonao gera ciclos pequenos (isto eh, nao ha um grupinho de pessoas quetrocam presentes entre si), que costumam tumultuar a mecanica dadistribuiao de presentes e eh facilmente implementado computacionalmente(basta gerar numeros aleatorios ; quem recebe o menor eh o primeiroetc.) e evita falsas meladas de sorteio (em sorteios feitos compapeizinhos, eh comum quem sorteia o mala do grupo dizer que sorteou asi mesmo).Alem disso, sorteios com papeizinhos so tem cerca de 36% deprobabilidade de darem certo (isto eh, de nao haver um cara que sorteoua si mesmo). A esse respeito leia um artigo do Gugu na RPM de cujonumero nao re
cordo agora, mas que alguem certamente indicarah.Gabriel Prgola wrote:

  Boa tarde,Estavamos pensando em um amigo secreto aqui na minha repblica, mas o
  
  
  nmero
  

  de pessoas que moram aqui  mpar, logo, pensamos em chamar mais uma
  
  
  pessoa
  

  para que desse certo.Mas depois pensei direito e vi que  possvel a realizao perfeita daconfraternizao com um nmero mpar de pessoas.Por exemplo: trs pessoas participando, A, B e CA tira BB tira CC tira AE vi que no importa o nmero de pessoas.S no consegui achar uma explicao matemtica para este fato.Algum poderia me dar uma explicao do porqu disto?Abraos,Gabriel=Instrues para entrar na lista, sair da lista e usar a lista emhttp://www.mat.puc-rio.br/~nicolau/olimp/obm-l.htmlO administrador desta lista  [EMAIL PROTECTED]=
  
  =Instrues para entrar na lista, sair da lista e usar a lista emhttp://www.mat.puc-rio.br/~nicolau/olimp/obm-l.htmlO administrador desta lista  [EMAIL PROTECTED]=
  
  =Instrues para entrar na lista, sair da lista e usar a lista emhttp://www.mat.puc-rio.br/~nicolau/olimp/obm-l.htmlO administrador desta lista  [EMAIL PROTECTED]=
  
  
  
  


Re: [obm-l] Amigo secreto...

2002-12-05 Por tôpico Augusto César Morgado



Pensemos na lista. Suponhamos que haja x participantes e que foi feito um
sorteio valido. Qual eh a prob. de Andre T tirar Dirichlet e vice-versa?
A prob. de Andre T tirar Dirichlet eh 1/(n-1) e, depois disso, a prob. de
Dirichlet tirar Andre T eh 1/(n-1). A resposta eh 1/[n-1)^2]

JOO CARLOS PAREDE wrote:

  Acompanhei a discusso a respeito do amigo secreto, at onde pude. 
  Lembrei-me de um problema surgido com a minha noiva em seu servio. 
  O sorteio do amigo secreto  feito em junho e no final do ano so entregue
os presentes. Durante o semestre h um mural onde se colocam mensagens para
o amigo secreto, identificando-o a partir de um pseudonimo que  combinado
na hora do sorteio. 
  Ela tirou a diretora da instituio onde ela trabalha (descobri, perguntando
o pseudonimo da diretora). A o que eu fiquei pensando  o seguinte: 
  Sendo n o nmero de funcionrios, qual a probabilidade do PARTICIPANTEX
retirar o PARTICIPANTEY e vice-versa?? 
   
  Gabriel_Prgola [EMAIL PROTECTED] wrote: 
  
Boa tarde,

Estavamos pensando em um amigo secreto aqui na minha repblica, mas o nmero
de pessoas que moram aqui  mpar, logo, pensamos em chamar mais uma pessoa
para que desse certo.
Mas depois pensei direito e vi que  possvel a realizao perfeita da
confraternizao com um nmero mpar de pessoas.

Por exemplo: trs pessoas participando, A, B e C

A tira B
B tira C
C tira A

E vi que no importa o nmero de pessoas.
S no consegui achar uma explicao matemtica para este fato.

Algum poderia me dar uma explicao do porqu disto?


Abraos,
Gabriel


=
Instrues para entrar na lista, sair da lista e usar a lista em
http://www.mat.puc-rio.br/~nicolau/olimp/obm-l.html
O administrador desta lista  
=! 



JOO CARLOS PAREDE 


Busca Yahoo! 

 O melhor lugar para encontrar tudo o que voc procura na Internet 





Re: [obm-l] Amigo secreto...

2002-12-04 Por tôpico Augusto César Morgado
Um processo extremamente eficiente de fazer um sorteio de amigo oculto 
eh fazer uma permutaçao (isto eh, colocar os nomes das pessoas em fila) 
das pessoas. Ai o primeiro da fila presenteia o segundo, o segundo 
presenteia o terceiro,..., o ultimo presenteia o primeiro. Tal processo 
nao gera ciclos pequenos (isto eh, nao ha um grupinho de pessoas que 
trocam presentes entre si), que costumam tumultuar a mecanica da 
distribuiçao de presentes e eh facilmente implementado computacionalmente
(basta gerar numeros aleatorios ; quem recebe o menor eh o primeiro 
etc.) e evita falsas meladas de sorteio (em sorteios feitos com 
papeizinhos, eh comum quem sorteia o mala do grupo dizer que sorteou a 
si mesmo).
Alem disso, sorteios com papeizinhos so tem cerca de 36% de 
probabilidade de darem certo (isto eh, de nao haver um cara que sorteou 
a si mesmo). A esse respeito leia um artigo do Gugu na RPM de cujo 
numero nao recordo agora, mas que alguem certamente indicarah.

Gabriel Pérgola wrote:

Boa tarde,

Estavamos pensando em um amigo secreto aqui na minha república, mas o número
de pessoas que moram aqui é ímpar, logo, pensamos em chamar mais uma pessoa
para que desse certo.
Mas depois pensei direito e vi que é possível a realização perfeita da
confraternização com um número ímpar de pessoas.

Por exemplo: três pessoas participando, A, B e C

A tira B
B tira C
C tira A

E vi que não importa o número de pessoas.
Só não consegui achar uma explicação matemática para este fato.

Alguém poderia me dar uma explicação do porquê disto?


Abraços,
Gabriel


=
Instruções para entrar na lista, sair da lista e usar a lista em
http://www.mat.puc-rio.br/~nicolau/olimp/obm-l.html
O administrador desta lista é [EMAIL PROTECTED]
=





=
Instruções para entrar na lista, sair da lista e usar a lista em
http://www.mat.puc-rio.br/~nicolau/olimp/obm-l.html
O administrador desta lista é [EMAIL PROTECTED]
=



Re: [obm-l] ab

2002-12-02 Por tôpico Augusto César Morgado



As vezes uma soma de 3 parcelas menores que 1 dah maior que 1.

Johann Peter Gustav Lejeune Dirichlet wrote:

   
  Juliana Freire [EMAIL PROTECTED] wrote: 
  
  


Tem alguma coisa errada neste enunciado.
Por exemplo, se a = b= c = 1/3, 
a^2b + b^2c + c^2a = 3^(1/3) ~ 1.44(desde quando?Se o cara de la
e menor que 1...)


- Juliana

  
  
  Poderia ajudar nessa questo:
Sejam a, b e c pertencentes ao reais positivos tais que 
a+b+c=1. Prove que a^2b + b^2c + c^2a  4/27
  
  
  


__
Venha para a VilaBOL!
O melhor lugar para voc construir seu site. Fcil e grtis!
http://vila.bol.com.br



=
Instrues para entrar na lista, sair da lista e usar a lista em
http://www.mat.puc-rio.br/~nicolau/olimp/obm-l.html

O administrador desta lista  
[EMAIL PROTECTED]

=





Yahoo! Acesso
Grtis

 Internet rpida, grtis e fcil. Faa o download do discador agora mesmo. 






[obm-l] Re:

2002-12-02 Por tôpico Augusto César Morgado
Os números que possuem uma quantidade impar de divisores sao os 
quadrados perfeitos!
A resposta eh 31.

Roberto Gomes wrote:

Quantos números de 1 a 1000 possuem números impar de divissores




=
Instruções para entrar na lista, sair da lista e usar a lista em
http://www.mat.puc-rio.br/~nicolau/olimp/obm-l.html
O administrador desta lista é [EMAIL PROTECTED]
=



Re: [obm-l] ab

2002-11-29 Por tôpico Augusto César Morgado



Acho que eh
(a^2)b + (b^2)c + (c^2)a 4/27

Juliana Freire wrote:

  
  
  Tem alguma coisa errada neste enunciado.
  Por exemplo, se a = b= c = 1/3, 
  a^2b + b^2c + c^2a = 3^(1/3) ~ 1.44
  
  
  - Juliana
  


Poderia ajudar nessa questo:
Sejam a, b e c pertencentes ao reaispositivos tais que 
a+b+c=1. Prove que a^2b + b^2c + c^2a 4/27



  

__
Venhapara a VilaBOL!
O melhor lugar para voc construir seu site. Fcil egrtis!
  http://vila.bol.com.br
  
  
  
=
Instruespara entrar na lista, sair da lista e usar a lista em
  http://www.mat.puc-rio.br/~nicolau/olimp/obm-l.html
  
Oadministrador desta lista  
[EMAIL PROTECTED]

=
  
  
  
  
  
  


Re: [obm-l] ajuda

2002-11-29 Por tôpico Augusto César Morgado



No.
Retas cortam essa curva em geral em 3 pontos (eventualmente imaginarios).
As tangentes a cortam em dois pontos coincidentes (ou seja, o ponto de tangncia)
e em um outro.
Por exemplo, a tangente em x=1 eh y = 2x - 2
Resolvendo y = 2x - 2 , y = x^3 - x encontramos uma raz dupla x=1 e uma
simples x = -2
Por exemplo, a tangente em x=0 eh y = -x.
Resolvendo y = -x, y = x^3 - x encontramos uma raz tripla x=0

Fabio Dias Moreira wrote:

  On Wed, Nov 27, 2002 at 09:27:23AM -0200, Augusto Csar Morgado wrote:
  
[...]Uma soluao sem derivada seria:Achando a interseao da reta com a curva,obtemos a equaaox^3 - x = 2x + nx^3 -3x - n = 0Esta equaao deve ter raiz dupla.[...]

No  necessrio, antes, garantir que qualquer tangente  curva (y = x^3 - x) a intersecta em no mximo um ponto (ou algum resultado do gnero)? Estamos descartando as tangentes que cortam a curva em dois ou mais pontos.[]s,






Re: [obm-l] da Vunesp

2002-11-28 Por tôpico Augusto César Morgado
D
f sobrejetiva significa precisamente isto: para todo a pertencente a B, 
a equação f(x) = a tem soluçao x pertencente a A. O fato de f ser 
injetiva significa que f(x) = a não pode ter mais de uma soluçao.

pichurin wrote:

Sejam A e B dois conjuntos não vazios  tais que
a sua intersecção é o conjunto vazio e seja f de A em
B uma função injetora.Se a é um elemento de B então,
para x pertencente a A, a equação f(x)=a
a) Não tem solução
b)tem duas soluções
c) tem umaúnica solução
d) terá solução se a função f for sobrejetora
e) tem mais que duas soluções.



___
Yahoo! Acesso Grátis
Internet rápida, grátis e fácil. Faça o download do discador agora mesmo.
http://br.acesso.yahoo.com/
=
Instruções para entrar na lista, sair da lista e usar a lista em
http://www.mat.puc-rio.br/~nicolau/olimp/obm-l.html
O administrador desta lista é [EMAIL PROTECTED]
=





=
Instruções para entrar na lista, sair da lista e usar a lista em
http://www.mat.puc-rio.br/~nicolau/olimp/obm-l.html
O administrador desta lista é [EMAIL PROTECTED]
=



Re: [obm-l] AJUDA

2002-11-27 Por tôpico Augusto César Morgado



Os angulos dos setores sao proporcionais aos valores das grandezas e somam
360 graus.
Sao, portanto e em graus e aproximadamente:
885/3201 * 360 = 100
868/3201 * 360 = 98
714/3201 * 360 = 80
444/3201 * 360 = 50
290/3201 * 360 = 33
A ideia eh consierar 33 como x e aih as medidas sao, aproximadamente 3x,
3x, 5x/2, 3x/2 e x.
Espero ter ajudado.
Morgado
PS: Voce nao perguntou, mas nao resisto ah tentaao de dizer que poucas vezes
vi uma questao tao idiota como esta, ate mesmo na ausencia do respectivamente
no enunciado, na utilizaao indevida da palavra conjunto no enunciado, na
desconexo entre as alternativas etc. 

Margarida Lanna wrote:

  
  
  
  Esta questo  do exame de seleo do
Coltec -  BH.
  Quem puder, resolva-a, por favor.
  
  Foi dada uma tabela e, desta tabela foram
tirados  alguns numeros. O enunciado foi:
  
  Deseja-se representar as quantidades inteiras
de  contos de ris das receitas em 1849 que so: 885, 868, 714, 444 e 290,
conforme  os dados da tabela 2, em um grfico de setores. Sendo X uma medida
em graus, o  conjunto de medidas mais coerente com o conjunto de medidas
dos ngulos dos  setores desse grfico ser:
  
  A resposta  letraC
  
  A) { X + 60 , X + 50, X + 40 , X +
30, x +  20}
  B) { X / 5 , X ,  X + 20, X - 10 ,
2X  }
  C) { 3X , 3X , 5X/2, 3X/2 ,  X}
  D) { X/4, 2X, X, X+ 20, x - 30 }
  
  Abraos, Margarida  Lanna
  
  
  
  
  


Re: [obm-l] ajuda

2002-11-27 Por tôpico Augusto César Morgado



Seja (x,y) o ponto de tangencia. A derivada nesse ponto eh o coeficiente
angular da tangente, 2
Temos o sistema
y = x^3 - x
y = 2x + n
3x^2 - 1 = 2
Resolvendo, ha duas soluoes
x = 1 y=0  n = -2
x = -1 y=0  n=2
Uma soluao sem derivada seria:
Achando a interseao da reta com a curva,obtemos a equaao 
x^3 - x = 2x + n
x^3 -3x - n = 0
Esta equaao deve ter raiz dupla.
As raizes serao  a, a, b
2a+b = 0
a^2 + 2ab = -3
a^2 b = n 
Resolvendo,
a =1  b= -2 n = -2
a = -1 b=2   n = 2
[EMAIL PROTECTED] wrote:

Dada a curva y = x - x e a reta tangente y = 2x + n , determine o valor
de n.
  
  
  


Re: [obm-l] Integrais

2002-11-26 Por tôpico Augusto César Morgado



Na primeira, vamos supor C positivo (Se C for negativo dah completamente
diferente). Para aliviar a notaao vamos chamar de a a raiz quadrada
positiva de C, C = a^2.
Faa a substituiao x = a tan z
A integral se transforma em Integral dea [(tanz)^2 / secz]
a (secz)^2 dz = 
Int (a^2) (tanz)^2 (secz) dz
Usando (tanz)^2 = (secz)^2 -1, voce recairah em duas integrais que, postas
constantes em evidncia, sao:
i) Int secz dz 
Esta eh imediata, dah ln Modulo (secz + tan z)
ii) Int (secz)^3 
Esta eh chata, tem que integrar duas vezes por partes
Na primeira integraao voce bota u = secz,  dv = (secz)^2 dz
Dah secz . tanz - Int secz (tanz)^2 dz 
Aqui voce faz (tanz)^2 = (secz)^2 -1 e vai cair novamente na integral de
(secz)^3
Voce estara diante de uma equaao do primeiro grau em integral de (secz)^3
. 
Marcos Reynaldo wrote:

  Ah!! Mas que belo digitador eu sou! Do jeito queenviei est fcil. Faltou as raizes quadradas.Ai vai a verso corrigida.1) int(x^2/sqrt(x^2+ C))dx2) int(sqrt(8x^2+6x+5))dxAgora sim. --- Augusto Csar Morgado [EMAIL PROTECTED]escreveu:  2) 8(x^3)/3  + 3(x^2) + 5x + C
  
1) x^2/ (x^2+a) = 1 - a/(x^2+a)A integral dah  x - (raiz de a)  arctan (x/raiz dea) + C, supondo a positivoMarcos Reynaldo wrote:

  Ol colegas!Estava tentando resolver algumas integrais mas,
  
  travei
  
nas duas que seguem. Alguma dica ??1) int(x^2/(x^2 + C) dx   (onde C  uma constante)2) int(8x^2+6x+5)dxObrigado.Marcos.


___Yahoo! Acesso GrtisInternet rpida, grtis e fcil. Faa o download do discador agora mesmo.http://br.acesso.yahoo.com/=Instrues para entrar na lista, sair da lista e usar a lista emhttp://www.mat.puc-rio.br/~nicolau/olimp/obm-l.htmlO administrador desta lista  [EMAIL PROTECTED]=






Re: [obm-l] Integrais

2002-11-26 Por tôpico Augusto César Morgado



8x^2 + 6x + 5 = 8(x+ 3/8)^2 + 31/8
Chame 2sqrt2 (x+3/8) de sqrt (31/8) tanz
Cai na mesma integral de (secz)^3.

Marcos Reynaldo wrote:

  Ah!! Mas que belo digitador eu sou! Do jeito queenviei est fcil. Faltou as raizes quadradas.Ai vai a verso corrigida.1) int(x^2/sqrt(x^2+ C))dx2) int(sqrt(8x^2+6x+5))dxAgora sim. --- Augusto Csar Morgado [EMAIL PROTECTED]escreveu:  2) 8(x^3)/3  + 3(x^2) + 5x + C
  
1) x^2/ (x^2+a) = 1 - a/(x^2+a)A integral dah  x - (raiz de a)  arctan (x/raiz dea) + C, supondo a positivoMarcos Reynaldo wrote:

  Ol colegas!Estava tentando resolver algumas integrais mas,
  
  travei
  
nas duas que seguem. Alguma dica ??1) int(x^2/(x^2 + C) dx   (onde C  uma constante)2) int(8x^2+6x+5)dxObrigado.Marcos.


___Yahoo! Acesso GrtisInternet rpida, grtis e fcil. Faa o download do discador agora mesmo.http://br.acesso.yahoo.com/=Instrues para entrar na lista, sair da lista e usar a lista emhttp://www.mat.puc-rio.br/~nicolau/olimp/obm-l.htmlO administrador desta lista  [EMAIL PROTECTED]=






Re: [obm-l] ajuda

2002-11-25 Por tôpico Augusto César Morgado



Olha, nao posso dara resposta que desejaria. Mas se voce entrar no google
com AHSME e AIME voce encontrara muita coisa. O chato eh que a lista de
endereos que aparece eh enorme e muitos deles tem apenas algumas poucas
questoes como exemplos.
A MAA publicou 5 (6?) livros com essas questes. Chamam-se The Contest Problem
Book.
Morgado

Daniel Pini wrote:

  
  
  alguem sabe um site aonde eu poderia encontrar
 questes de matematica das timas competies americanas: AHSME e  AIME?
  
  
  
  


Re: [obm-l] Integrais

2002-11-25 Por tôpico Augusto César Morgado
2) 8(x^3)/3  + 3(x^2) + 5x + C
1) x^2/ (x^2+a) = 1 - a/(x^2+a)
A integral dah  x - (raiz de a)  arctan (x/raiz de a) + C, supondo a 
positivo

Marcos Reynaldo wrote:

Olá colegas!

Estava tentando resolver algumas integrais mas, travei
nas duas que seguem. Alguma dica ??

1) int(x^2/(x^2 + C) dx   (onde C é uma constante)

2) int(8x^2+6x+5)dx

Obrigado.

Marcos.

___
Yahoo! GeoCities
Tudo para criar o seu site: ferramentas fáceis de usar, espaço de sobra e acessórios.
http://br.geocities.yahoo.com/
=
Instruções para entrar na lista, sair da lista e usar a lista em
http://www.mat.puc-rio.br/~nicolau/olimp/obm-l.html
O administrador desta lista é [EMAIL PROTECTED]
=





=
Instruções para entrar na lista, sair da lista e usar a lista em
http://www.mat.puc-rio.br/~nicolau/olimp/obm-l.html
O administrador desta lista é [EMAIL PROTECTED]
=



Re: [obm-l] Matriz Inversa

2002-11-24 Por tôpico Augusto César Morgado



Daniel, ha um teorema (chamado de teorema de Binet-Cauchy) que diz que det(AB)
= detA*detB (A e B quadradas de mesmo tamanho, eh claro).
A sua hipotese AX = I implica det(AX) = detI , 
detA* detX =1
e, portanto, detA e detX sao ambos diferentes de zero.
Em suma, a sua hipotese AX=I com A e X quadradas fe mesma ordem assegura
que as duas tem determinantes diferentes de zero e, portanto, que A e X sao
invertiveis.
Dai, AX =I , 
(A^-1)AX = (A^-1)I
IX=(A^-1)
X = A^-1
A resposta a sua pergunta eh SIM. ( e nao precisa nem saber que detA eh diferente
de zero, isso eh consequencia de AX=I e A quadrada.
Morgado
Daniel wrote:

  - Original Message -From: Augusto Csar Morgado [EMAIL PROTECTED]To: [EMAIL PROTECTED]Sent: Saturday, November 23, 2002 10:13 PMSubject: Re: [obm-l] Matriz Inversa
  
Daniel,em principio voce deve verificar as duas coisas pois, por definiao, Xeh a inversa de A significaAX = XA = I .Mas , vale o teorema: Se A eh quadrada e AX = I, entao XA=ILogo, por causa desse teorema, basta verificar uma so das duas coisas.A prova do teorema eh simples.Se AX=I, det(AX) = detI,   detA . detX = 1,  detA diferente dezero,A eh invertivel.

*Prof Morgado,Na linha acima no  preciso saber que det X  diferente de zero?Pois como havia dito no se sabe nada sobre a matriz X, apenas que ela quadrada de mesma ordem que A. Minha pergunta : dado o produto de matrizesquadradas de mesma ordem AX = I, sabendo que det A  diferente de zero, eno sabendo nada sobre o det X, X  necessriamente a inversa de A?Obrigado pela Anteno, desculpe pela instistnciaDaniel O. Costa=Instrues para entrar na lista, sair da lista e usar a lista emhttp://www.mat.puc-rio.br/~nicolau/olimp/obm-l.htmlO administrador desta lista  [EMAIL PROTECTED]=






Re: [obm-l] dificuldade

2002-11-24 Por tôpico Augusto César Morgado



Um polinomio eh f(x) = x^2. Nao tah faltando nada no enunciado?

[EMAIL PROTECTED] wrote:

Sabendo que para todo x pertencente aos reais tem-se P(x) = P(-x-1). Determine
 um polinmio f(x) tal que P(f(x)) = P(f(-x)).
  
  
  


Re: [obm-l] Matriz Inversa

2002-11-24 Por tôpico Augusto César Morgado



AX=I significa explicitamente que A tem inversa a direita.
AX=I nao significa, nem implicitamente que A eh invertivel. Por exemplo,
considere A 1x2 com elementos 1 e 2 e considere X 2x1 com elementos 3 e
-1. AX=I e A nao eh invertivel, isto eh, nao existe Y tal que YA=I.
Agora, conforme provei em outra mensagem, A quadrada e AX=I implica XA=I
e, portanto, X eh a inversa de A. 
Ha que provar as coisas, nao?
Domingos Jr. wrote:

   Carissimos, voces estao supondo muito mais coisas do que o Daniel: o
  
Daniel supunha apenas A quadrada e com inversa a direita. Laurito estahsupondo que A tem inversa a direita e tem inversa a esquerda. Domingos,que A eh invertivel.Morgado

Prof., se o enunciado nos diz que existe X tal que A.X = I ele estafirmando implicitamente que A possui inversa, no?=Instrues para entrar na lista, sair da lista e usar a lista emhttp://www.mat.puc-rio.br/~nicolau/olimp/obm-l.htmlO administrador desta lista  [EMAIL PROTECTED]=






Re: [obm-l] Matriz Inversa

2002-11-23 Por tôpico Augusto César Morgado
Daniel,
em principio voce deve verificar as duas coisas pois, por definiçao, X 
eh a inversa de A significa
AX = XA = I .
Mas , vale o teorema: Se A eh quadrada e AX = I, entao XA=I 
Logo, por causa desse teorema, basta verificar uma so das duas coisas.
A prova do teorema eh simples.
Se AX=I, det(AX) = detI,   detA . detX = 1,  detA diferente de 
zero,A eh invertivel.
Chame de B a inversa de A
AX = I ,   BAX = BI, IX = B,   X=B
Logo, X eh a inversa de A.
Eh essencial que A seja quadrada. Se A nao for quadrada, pode ser 
possivel encontrar B tal que AB=I e BA diferente de I.

Daniel wrote:

   Olá à todos os membros da lista!

   Uma pergunta teórica sobre matrizes:

   Sejam A e X matrizes quadradas de ordem n e I a
matriz identidade de mesma ordem. Para a equação:
   AX = I, posso afirmar que X é a inversa de A, ou
é preciso definir que
   AX = XA = I

   Grato

   Daniel O . Costa

=
Instruções para entrar na lista, sair da lista e usar a lista em
http://www.mat.puc-rio.br/~nicolau/olimp/obm-l.html
O administrador desta lista é [EMAIL PROTECTED]
=





=
Instruções para entrar na lista, sair da lista e usar a lista em
http://www.mat.puc-rio.br/~nicolau/olimp/obm-l.html
O administrador desta lista é [EMAIL PROTECTED]
=



Re: [obm-l] PAs de ordens1

2002-11-22 Por tôpico Augusto César Morgado
somatorio de [(k+1)^3 - k^3] = somatorio de 3k^2 - somatorio de 3k + 
somatorio de 1
Se os somatorios sao com k variando de 1 ate n, obtem-se
(n+1)^3 - 1 = 3 somatorio de k^2 - 3 somatorio de k + n
somatorio de k eh soma de PA, dah n(n+1)/2
Fazendo as contas dah somatorio de k^2 = n(n+1)(2n+1)/6
Essas coisas encontram-se no Progressoes e Matematica Financeira da SBM 
(na quarta ediçao ha soluçoes delineadas) e no Analise Combinatoria e 
Probabilidade (capitulo 4) ha uma tecnica para isso usando o Teorema das 
Colunas.
Se voce quiser aproveitar para aprender mais, vale a pena ler (nao eh um 
livro grande, embora seja um grande livro) o livro de diferenças finitas 
do Richardson, An introduction to the calculus of finite differences.
Morgado.

Alexandre Tessarollo wrote:

  Estou num momento de diarréia mental. Qual é e como deduzir a fórmula de somatório de x^2, para x=1,2,..,n? 

  Ou, mais genericamente, como se calcula a soma do n primeiros termos de uma PA de 2a ordem, onde b[n+1]-b[n]=a[n], sendo a[n] o termo de uma PA normal(de 1a ordem)? Naturalmente temos a[1], R e b[1].

  Generalizando ainda mais, sejam a{1}[1], a{2}[1],..,a{k}[1] respectivamente os primeiros termos de PAs de 1a, 2a,..,k-ésima ordem e R a razão da PA de primeira ordem. Em função desses parâmetros, qual a soma dos n primeiros termos da PA de k-ésima ordem?

[]'s

Alexandre Tessarollo



=
Instruções para entrar na lista, sair da lista e usar a lista em
http://www.mat.puc-rio.br/~nicolau/olimp/obm-l.html
O administrador desta lista é [EMAIL PROTECTED]
=



Re: [obm-l] PAs de ordens1

2002-11-22 Por tôpico Augusto César Morgado



Dah n(n+1)(2n+1)/6
Morgado

Olimpiada Brasileira de Matematica wrote:

  At 09:55 AM 11/22/02 -0300, you wrote:
  

 Estou num momento de diarria mental. Qual  e como deduzir a frmula
  
  de somatrio de x^2, para x=1,2,..,n?
  
  Vc pode tentar fazer perturbacao no somatorio dos cubos, vejan n-1  1+ sum(k^3) = sum(k^3) + n^3   k=2k=1 Agora devemos alterar o somatorio para ficar com os mesmos indicesn-1 n-11+ sum(k+1)^3 = sum(k^3) + n^3   k=1  k=1   n-1 n-1n-1 n-1 n-11+ sum(k^3) +3.sum(k^2)+3.sum(k)+ sum(1)= sum(k^3) + n^3   k=1 k=1k=1 k=1 k=1agora cancela o somatorio de k^3 (tchu), e voc fica com  n-13.sum(k^2)=n^3-n-3.n(n-1)/2  k=1agora eh s trabalhar o lado direito que vc achan(n+1)(2n-1)/6espero nao ter errado em conta abracosMarcelo
  
=Instrues para entrar na lista, sair da lista e usar a lista emhttp://www.mat.puc-rio.br/~nicolau/olimp/obm-l.htmlO administrador desta lista  [EMAIL PROTECTED]=

=Instrues para entrar na lista, sair da lista e usar a lista emhttp://www.mat.puc-rio.br/~nicolau/olimp/obm-l.htmlO administrador desta lista  [EMAIL PROTECTED]=






Re: [obm-l] Soma de compostos(p/rocha31),TRIGONOMETRIA e areas e volumes

2002-11-21 Por tôpico Augusto César Morgado



 fcil.  s transformar uns produtos em somas e umas somas em produtos.
No final h trs tipos de solues:
a) sen (30+x) = 0
b) cos 2x = 1/2
c) cos (30 + 3x) = 0
Se eu no errei alguma conta!
Confira a! 
Morgado

Johann Peter Gustav Lejeune Dirichlet wrote:

  
   Ola turma
da Lista OBMEstou com uma resposta ao e-mail de rocha31:demonstre que
todo inteiro positivo maior que 11 pode ser escrito como soma de 2 compostos.
  SOLUAO:veja que paraospares naoi se faz muito:x+4
e par se e somente se x tambem o for.E x+411se e so se x52,logo
x e par maior que 2,logo e composto.Pronto:x e 4 quebram x+4 comoqueremos(caso
x+4 par).
  Vamos verse com os impares da certo.9 e o menor composto
impar.E x+9 e impar se e so se x for par.E x+911,x2,logo (de novo!)x
e par maior que 2,logo e composto.Pronto:x e9 quebram x+9 comoqueremos(caso
x+9 impar).Gostou?Ate que nao era dificil mas tem que ter uma imaginaao...
  -
  Agora e a minha parte!
  Estou tentando resolver uma equaao trigonometrica mas
nao sei como me destrinchar dos carinhas:
  sen(30+2x)*cos(2x)*sen(60+2x)=sen(30+x)*cos(30-x)*sen(30+4x).
  E tambem tenho uma duvida :alguem sabe onde eu posso
encontrar livros sobre o Teorema de Gelfond(aquele do a^b e transcedente
se a e b forem algebricos e b nao for racional(tipo i^i,2^(2^/0,5)),e
tambem definioes precisas sobre o que e area,comprimento e volume de figuras
quaisquer(por favor,sem apelar para Riemann...)
  Se alguem puder me ajudar,agradeo muito.
  Ass.:Johann
  
  
  
  TRANSIRE SVVM PECTVS MVNDOQUE POTIRE
  CONGREGATI EX TOTO ORBE MATHEMATICI OB SCRIPTA INSIGNIA TRIBVERE
  Fields Medal(John Charles Fields)
  
  
  Yahoo! GeoCities
  
 Tudo para criar o seu site: ferramentas fceis de usar, espao de sobra
e acessrios. 
  
  
  


Re: [obm-l] Mais ajuda!

2002-11-20 Por tôpico Augusto César Morgado



(1+i)^n = (1-i)^n equivale a [(1+i)/(1-i)]^n = 1
(1+i)/(1-i) = (1+i)^2/[(1+i)(1-i)] = (1+2i+i^2)/(1-i^2) = 2i/2 = i 
Logo, a pergunta eh quando que i^n = 1 .
A resposta eh n multiplo de 4.

Sharon Guedes wrote:

  Ol pessoal,
ser que algum poderia me ajudar nessas questes:
  1) Determine
o conjunto soluo da equao
 z  + z  z .`
z = 3 + 3i 
   
  Resposta : 3 + 3i
  2)Sabendo que z  um nmero complexo
tal quez . `z= 24 ,calcule
o mdulo de z.
  Resposta: 2
 6
  
  
   
  (UFRGS) A igualdade (1 + i)^n = (1-
i )^n se verifica se e somente se:
  
n = 4K, k 
 z 
n = 0
n  mpar
n  par
n  primo.
  
  At. Sharon.
  
Ol pessoal, ser que algum poderia me ajudar em mais essas questes:
  
1) Determine o conjunto soluo da equao
 z  + z  z .`
z = 3 + 3i 
  Resposta : 3 +
3i
  2)Sabendo que z
 um nmero complexo tal quez . `
z= 24 ,calcule o mdulo de z.
  Resposta: 2
 6
  
  
  (UFRGS) A igualdade
(1 + i)^n = (1- i )^n se verifica se e somente se:
  
n = 4K, k 
 z 
n = 0
n  mpar
n  par
n  primo.
  
  At. Sharon.
  
Ol pessoal, ser que algum poderia me ajudar nessas questes:
  
1) Determine o conjunto soluo da equao
 z  + z  z .`
z = 3 + 3i 
  
Resposta : 3 + 3i
  
2)Sabendo que z  um nmero complexo tal quez . 
`z= 24 ,calcule o mdulo de z.
  
Resposta: 2 6
  
 
  
(UFRGS) A igualdade (1 + i)^n = (1- i )^n se verifica se e somente se:
  

n = 4K, k  z 

n = 0

n  mpar

n  par

n  primo.
  
  At. Sharon.
  
  
  Yahoo! GeoCities
  
 Tudo para criar o seu site: ferramentas fceis de usar, espao de sobra
e acessrios. 
  
  
  


Re: [obm-l] z^z - mais perguntas

2002-11-20 Por tôpico Augusto César Morgado



Bem, essa histria de positivo e negativo, deriva da noo de positivo. Um
negativo  um nmero cujo simtrico  positivo.
E o que so nmeros positivos? Os positivos formam uma classe tal que: a
soma de positivos  positivo, o produto de positivos  positivo e (tricotomia),
dado um numero qualquer, vale uma e uma s das alternativas: ele  zero,
ele  positivo, ele  negativo. A partir da noo de positivo  que se definem
maior (a maior que b significa a menos b  positivo), menor...
Nos complexos, no existe uma classe de positivos com as propriedades
acima. Com efeito, como i no  zero, ou i  positivo ou
 negativo. Se i  positivo, i*i = -1  positivo. Absurdo Se i  negativo,
-i  positivo e -i * -i = -1  positivo. Absurdo.
Portanto, no h nos complexos uma ordem com as propriedades acima.
PS: -1 positivo  absurdo porque 1  positivo. E 1  positivo porque no
 zero e se fosse negativo, -1 seria positivo e -1*-1 = 1 seria positivo.

Jose Francisco Guimaraes Costa wrote:

  
  
  
Jose Francisco  Guimaraes Costa wrote:
 
 Sejam z1 e z2 dois nmeros  complexos.

A operao z1^z2  definida? Se for, qual  sua definio?
  
  
  On Mon, Nov 18, 2002 at 10:30:40AM
-0200,  Augusto Csar Morgado wrote:
 z1^z2 = exp (z2 * ln  z1)

  
  
  From
: "Nicolau C. Saldanha" 
[EMAIL PROTECTED]
  
  Date
: Mon, 18 Nov 2002 15:08:27 -0200 
  
A definio do Morgado  tima mas  preciso chamar a ateno  para
o fato de ln z1 no estar to bem definido assim. A funo ln  no
pode ser definida assim

ln : C - {0} -  C

precisamos fazer um corte, como por exemplo

ln :  C - {z in R, z = 0} - C

e escolhas diferentes do corte  produzem valores diferentes para ln z1.

[]s,  N.
  
  Mais perguntas:
  
  (1) 
Usando a mesma linguagem segundo aqual a expresso  
  
  A= sqrt(B)
  
   lida como "A  igual  raiz quadrada
de B",  como ler a expresso
  
  ln : C - {z in R, z = 0} -
C  ?
  
  (2) N diz "precisamos fazer um corte,
como  por exemplo ... ". Por que precisamos fazer um corte (ou por que "A
funo ln  no pode ser definida assim: ln : C - {0} - C") ?
  
  (3) A afirmao "precisamos fazer
um corte,  como por exemplo ... e escolhas diferentes do corte produzem valores
diferentes  para ln z" me deixa com a idia de que eu posso escolher o corte
que me convier,  o que faz com que a funo "ln z" no tenha uma definio
nica.  isso  mesmo?
  
  (4) Faz sentido dizer que um nmero
complexo   positivo ou negativo? Se fizer, quando ele  positivo e quando
  negativo?
  
  (5) Por favor sugiram livros onde
eu possa  encontrar respostas para este tipo de perguntas. Embora eu tenha
estudado  nmeros complexos e trabalhado com eles - sou engenheiro eletrnico
- no me  lembro de ter sido exposto s definies e conceitos acima.
  
  JF (Rio de Janeiro, iniciado na
cincia da  matemtica pelo mesmo Prof. Morgado que iniciou o Morgado um
ano depois de  mim)
  
  
  
  


Re: [obm-l] z^z - mais perguntas

2002-11-20 Por tôpico Augusto César Morgado



Um bom livro de Varivel complexa, que engenheiros e matemticos podem ler
com gosto,  o do R. Churchill. Aposto que depois dessa chovero mensagens
falando em livros mais "matemticos" como o do Ahlfors.
Morgado 

Jose Francisco Guimaraes Costa wrote:

  
  
  
Jose Francisco  Guimaraes Costa wrote:
 
 Sejam z1 e z2 dois nmeros  complexos.

A operao z1^z2  definida? Se for, qual  sua definio?
  
  
  On Mon, Nov 18, 2002 at 10:30:40AM
-0200,  Augusto Csar Morgado wrote:
 z1^z2 = exp (z2 * ln  z1)

  
  
  From
: "Nicolau C. Saldanha" 
[EMAIL PROTECTED]
  
  Date
: Mon, 18 Nov 2002 15:08:27 -0200 
  
A definio do Morgado  tima mas  preciso chamar a ateno  para
o fato de ln z1 no estar to bem definido assim. A funo ln  no
pode ser definida assim

ln : C - {0} -  C

precisamos fazer um corte, como por exemplo

ln :  C - {z in R, z = 0} - C

e escolhas diferentes do corte  produzem valores diferentes para ln z1.

[]s,  N.
  
  Mais perguntas:
  
  (1) 
Usando a mesma linguagem segundo aqual a expresso  
  
  A= sqrt(B)
  
   lida como "A  igual  raiz quadrada
de B",  como ler a expresso
  
  ln : C - {z in R, z = 0} -
C  ?
  
  (2) N diz "precisamos fazer um corte,
como  por exemplo ... ". Por que precisamos fazer um corte (ou por que "A
funo ln  no pode ser definida assim: ln : C - {0} - C") ?
  
  (3) A afirmao "precisamos fazer
um corte,  como por exemplo ... e escolhas diferentes do corte produzem valores
diferentes  para ln z" me deixa com a idia de que eu posso escolher o corte
que me convier,  o que faz com que a funo "ln z" no tenha uma definio
nica.  isso  mesmo?
  
  (4) Faz sentido dizer que um nmero
complexo   positivo ou negativo? Se fizer, quando ele  positivo e quando
  negativo?
  
  (5) Por favor sugiram livros onde
eu possa  encontrar respostas para este tipo de perguntas. Embora eu tenha
estudado  nmeros complexos e trabalhado com eles - sou engenheiro eletrnico
- no me  lembro de ter sido exposto s definies e conceitos acima.
  
  JF (Rio de Janeiro, iniciado na
cincia da  matemtica pelo mesmo Prof. Morgado que iniciou o Morgado um
ano depois de  mim)
  
  
  
  


Re: [obm-l] ---- Questão IME

2002-11-19 Por tôpico Augusto César Morgado



Epa! A pode no ser identicamente nula e A^3 = kA e A^2 diferente de kI.
Por exemplo, considere A 2x2 com primeira coluna 2 2  e segunda coluna
0 0. A no  identicamente nula, A^3 = 4A e A^2 no  igual a 4I.
Morgado


Salvador Addas Zanata wrote:

  Se A^3=kA, entao se A nao for identicamente nula, A^2=kI.Suponha que (A+I) nao seja inversivel. Entao o sistema (A+I)x=0 tem uma solucao x nao-identicamente nula.Assim, Ax=-x = A^2x=-Ax=xMas por outro lado, A^2x=kx, logo kx=x, absurdo pois x nao e identicamentenulo e k1.Abraco,SalvadorOn Tue, 19 Nov 2002, cfgauss77 wrote:
  
  Ficaria muito agradecido se algum me ajudasse na qusto do IME abaixo. -- Considere uma matriz A, nxn, de coeficientes reais, e k um nmero real diferente de 1. Sabendo que A^3=kA, prove que a matriz A+I  invertvel, onde I  a matriz identidade nxn. __Venha para a VilaBOL!O melhor lugar para voc construir seu site. Fcil e grtis!http://vila.bol.com.br=Instrues para entrar na lista, sair da lista e usar a lista emhttp://www.mat.puc-rio.br/~nicolau/olimp/obm-l.htmlO administrador desta lis
ta  [EMAIL PROTECTED]=

=Instrues para entrar na lista, sair da lista e usar a lista emhttp://www.mat.puc-rio.br/~nicolau/olimp/obm-l.htmlO administrador desta lista  [EMAIL PROTECTED]=






Re: [obm-l] ---- Questão IME

2002-11-19 Por tôpico Augusto César Morgado
Chame A+I de X.  
A = X - I.
Como A^3 = kA, fazendo as contas dá
X^3 - 3X^2 + 3X - I = kX - kI
X (X^2 - 3X +3I - kI) =  (1-k) I
A inversa de X é o produto do número 1/(1-k) pela matriz (X^2 - 3X +3I - 
kI).
Morgado
cfgauss77 wrote:

 Ficaria muito agradecido se alguém me ajudasse na 
qustão do IME abaixo.
-- Considere uma matriz A, nxn, de coeficientes reais, 
e k um número real diferente de 1. Sabendo que A^3=kA, 
prove que a matriz A+I é invertível, onde I é a matriz 
identidade nxn.


   


__
Venha para a VilaBOL!
O melhor lugar para você construir seu site. Fácil e grátis!
http://vila.bol.com.br


=
Instruções para entrar na lista, sair da lista e usar a lista em
http://www.mat.puc-rio.br/~nicolau/olimp/obm-l.html
O administrador desta lista é [EMAIL PROTECTED]
=




=
Instruções para entrar na lista, sair da lista e usar a lista em
http://www.mat.puc-rio.br/~nicolau/olimp/obm-l.html
O administrador desta lista é [EMAIL PROTECTED]
=



Re: [obm-l] z^z

2002-11-18 Por tôpico Augusto César Morgado



 z1^z2 = exp (z2 * ln z1)

Jose Francisco Guimaraes Costa wrote:

  
  
  Sejam z1 e z2 dois nmeros complexos.
  
  
  A operao z1^z2  definida? Se for, qual
sua  definio?
  
  JF
  
  
  
  


Re: [obm-l] Ajuda!

2002-11-18 Por tôpico Augusto César Morgado



3) (1-i) ^2 = 1 -2i + i^2 = 1 -2i -1 = -2i
(1-i)^4 = (-2i)^2 = 4(i^2) = -4
(1-i) ^12 = (-4)^3 = -64
(1-i) ^13 = (1-i)*(-64) = 64 ( -1 + i)
2) A resposta eh 1. O modulo de um complexo a+bi (raix quadrada de a^2+b^2)
) eh igual ao modulo do seu conjugado a - bi.
1) w^2 = cos 60 + i sen 60
Z^2 = cos 240 + i sen 240
W^2 + z^2 = 0
Sharon Guedes wrote:

  
Ol, pessoal! Ser que algum poderia me ajudar nestas questes:
(UFRGS) Se W = cos 30 + i . sen 30 e Z = cos 120 + i . sen 120, 
ento: 
a) W + Z = 0
b) W + Z = 0 
c) W - Z = 0 
d) W - Z = 0
  
e) W* - Z* = 0
  
(UFRGS) A razo entre o mdulo de um nmero complexo no nulo e o 
mdulo do seu conjugado : 
  
a) -2
  
b) -1
  
c)
  
d)1
  
e)2
  
(Provo99) Dado o nmero complexo Z = 1 - i, Z 
igual a:
  
a) 2 ( 1 - i )
  
b) 32 ( 1 + i )
  
c) 64 ( - 1 + i ) 
  
d) 13 raiz de 2 ( 1 - i )
  
e) 32 raiz de 2 ( - 1 - i )
  
At. Sharon.
  
  
  
  
  Yahoo! GeoCities
  
 Tudo para criar o seu site: ferramentas fceis de usar, espao de sobra
e acessrios. 
  
  
  


Re: [obm-l] Equaçao aberta

2002-11-14 Por tôpico Augusto César Morgado





felipe mendona wrote:
[EMAIL PROTECTED]">
  
   S = (2^0).(n-1)+(2^1).(n-2)+.+[2^(n-3)].2+[2^(n-2)].1
  
  
Vamos considerar a soma auxiliar T,
  [EMAIL PROTECTED]">

 T = S/[2^(n-2)] = (n-1)[0,5^(n-2)] + (n-2)[0,5^(n-3)] + ... +
2[0,5^1] + 1 


T = (n-1)[x^(n-2)] + (n-2)[x^(n-3)] + ... + 2[x^1] + 1 , para x=0,5.
[EMAIL PROTECTED]">
  
  T = derivada de [x^(n-1)] + [x^(n-2)] + ... + [x^2] + x +1 
= derivada de [(x^n - 1)/(x-1)] =[(x-1)n(x^(n-1))- (x^n - 1)]  / (x-1)^2
para x=0,5.
  
  
T=[ - (n+1) (0,5)^n + 1] / [0,5^2] = 4 - (n+1) [0,5^(n-2)]
  [EMAIL PROTECTED]">

S = 4* [2^(n-2)] - (n+1)   
Pessoal,existe uma forma fechada da expressao aberta (2^0).(n-1)+(2^1).(n-2)+.+[2^(n-3)].2+[2^(n-2)].1
?
 
 Aguardo respostas 
 Felipe
Mendona Vitria-ES.
 


MSN Messenger: converse com os seus amigos online. 
Instale grtis. Clique aqui.
 = 
Instrues para entrar na lista, sair da lista e usar a lista em http://www.mat.puc-rio.br/~nicolau/olimp/obm-l.html 
O administrador desta lista   = 
  





Re: [obm-l] probabilidade

2002-11-14 Por tôpico Augusto César Morgado



Pense nos 6 lugares _ _ _ _ _ _ , os dois primeiros formando o primeiro barco...
O numero de casos possiveis eh 6x5x4x3x2x1 = 720 (ha 6 lugares para colocar
A, 5 para B,)
O numero de casos favoraveis eh 6x1x4x1x2x1=48 (ha 6 lugares para colocar
A, 1 para B pois B tem que ficar no mesmo barco que A )
A resposta eh 48/720 = 1/15 

Juliana Lff wrote:
001601c28c32$d48589d0$c30a@wsjujuba">
  
  
  
Eu no  sei nada de probabilidade, se algum puder me ajudar, agradeo  muito!
  
  
  
Seis pessoas, A, B, C, D, E e F, vo atravessar  um rio em 3 horas. Distribuindo-se
ao acaso as pessoas de modo que fiquem duas  em cada barco, a probabilidade
de A atravessar junto com B, C junto com D e E  junto com F, :
  a) 1/5
b) 1/10
c) 1/15
d) 1/20
e)  1/25
  
  
  
  
  


Re: [obm-l] equação

2002-11-13 Por tôpico Augusto César Morgado



a = 1   b = 2i  c = 2 - 4i
b^2 - 4ac = 4 (i^2) - 4*1*(2 - 4i) = -4 - 8 + 16i = -12 + 16i = 4 ( -3 +
4i) = 4 [(1+2i)^2] = 
= [2(1+2i)]^2 = (2+4i)^2
As razes sao [ -2i + (2+4i)] / 2 = 1 + i   e [ -2i - (2+4i)] / 2 =
-1 - 3i 
Agora,  ruim de achar onde voce esta errando se voce nao manda sua soluao.
Morgado

[EMAIL PROTECTED] wrote:
[EMAIL PROTECTED]">
Determine as razes de z^2+2iz+2-4i=0 sendo i a unidade imaginria. No gabarito
d
 1+i e -1-3i como solues e verifica-se que  verdade...mas no brao d
respostas diferentes ...onde estou errando??
  Um abrao e um antecipado agradecimento a quem puder elucidar minha
duvida.
  Korshini,
  
  
  


Re: [obm-l] Editora Mir

2002-11-13 Por tôpico Augusto César Morgado



Insisto (estou desenvolvendo um problema grave de auto-estima; ningum l
o que eu escrevo!) que mensagens como esta deveriam vir acompanhadas da cidade
do remetente!
Morgado

Renato Lira wrote:
001f01c28b45$788854c0$306ef9c8@ig">
  
  
   Algum poderia me sugerir nomes  de
livros(bons para quem quer ITA e IME por ex)de uma editora russa  chamada
Mir? J ouvi falar muito bem de seus livros.
  
  
  
  


Re: [obm-l] subconjuntos

2002-11-12 Por tôpico Augusto César Morgado



Como nao sao disjuntos, se a interseao eh vazia? Sao disjuntos, sim.

Marcos Aurelio Almeida da Silva wrote:
[EMAIL PROTECTED]">
  mas a voc t contado o par {},{}, que no entra na contagem pois no um par de conjuntos disjuntos:. Marcos Aurlio Almeida da Silva.:..:. e-mail:   [EMAIL PROTECTED] .:..:. site  :   http://cin.ufpe.br/~maas .:.On Mon, 11 Nov 2002, Augusto Csar Morgado wrote:
  
A resposta  a metade de (3^n +1).Marcos Aurelio Almeida da Silva wrote:

  pessoal desculpe mas essa resposta est errada, pois havero 3^n relaespossveis s que algumas delas so equivalentes...acho que d para ficar assim:como {(x,1),(y,2)...}  equivalente a {(x,2),(y,1),...}, logo para todarelao existe uma outra completamente equivalente  ela,fica1. 3^n-1: exclui o caso em que todos so relacionados ao zero, noformando conjuntos disjuntos.2. (3^n-1)/2: exclui todas as relaes equivalenteslogo N = (3^n-1)/2acho que dessa vez t tudo ok:. Marcos Aurlio Almeida da Silva.:..:. e-mail:   [EMAIL PROTECTED] .:..:. site  :   http://cin.ufpe.br/~maas .:.On Mon, 11
 Nov 2002, Marcos Aurelio Almeida da Silva wrote:
  
bom, imagine um conjunto:A = {a1, a2, ..., an}imagine a seguinte relao que acossia a cada elemento do conjunto A umvalor:R: A - {0,1,2}vamos formar os seguintes conjuntos:B = { x / (x,1) pertence a R}C = { x / (x,2) pertence a R}D = { x / (x,0) pertence a R}logo temos dois conjuntos disjuntos que so subconjuntos de A (B e C),e o conjunto D que  formado pelos elementos que no entram em nenhum dosoutros dois conjuntos. Para contar o nmero de  subconjuntos disjuntos s contar o nmero de relaes, pois a cada par de subconjuntoscorresponde uma relao e a cada releo corresponde um par de conjuntos,logo a resposta deve ser 3^n..:. Marcos Aurlio Almeida da Silva.:..:. e-mail:   [EMAIL PROTECTED] .:..:. site  :   http://cin.ufpe.br/~maas .:.On Mon, 11 Nov 2002, cgmat wrote:

  Al pessoal, ser que algum poderia de dar uma dica na questo:De quantas formas podemos selecionar dois subconjuntos disjuntos  a partir de um conjunto finito com n elementos?Grato, C.Gomes.
  
  
  =Instrues para entrar na lista, sair da lista e usar a lista emhttp://www.mat.puc-rio.br/~nicolau/olimp/obm-l.htmlO administrador desta lista  [EMAIL PROTECTED]=
  
  
  
  =Instrues para entrar na lista, sair da lista e usar a lista emhttp://www.mat.puc-rio.br/~nicolau/olimp/obm-l.htmlO administrador desta lista  [EMAIL PROTECTED]=
  
  
  
  


Re: [obm-l] Dúvida

2002-11-12 Por tôpico Augusto César Morgado
O erro estah exatamente onde voce achou que estava. O professor nao deve 
ter dito o que o seu colega disse que ele disse.
Morgado

Marcos Reynaldo wrote:

Pessoal gostaria de uma ajuda para descobrir o erro da
seguinte sequencia:

16-36=25-45 -- 16-36+(9/4)=25-45+(9/4) --
(4-9/2)^2=(5-9/2)^2 -- 4-9/2=5-9/2 -- 4=5

Um colega me mostrou esse problema dizendo que foi
apresentado por seu professor. Fiquei confuso, pois
pensei o erro seria na hora de tirar o quadrado, aí
deveria ter o módulo, mas seu professor disse que não
era isso. Não consigo ver qual o erro então.

[]´s Marcos

___
Yahoo! GeoCities
Tudo para criar o seu site: ferramentas fáceis de usar, espaço de sobra e acessórios.
http://br.geocities.yahoo.com/
=
Instruções para entrar na lista, sair da lista e usar a lista em
http://www.mat.puc-rio.br/~nicolau/olimp/obm-l.html
O administrador desta lista é [EMAIL PROTECTED]
=





=
Instruções para entrar na lista, sair da lista e usar a lista em
http://www.mat.puc-rio.br/~nicolau/olimp/obm-l.html
O administrador desta lista é [EMAIL PROTECTED]
=



Re: [obm-l] subconjuntos

2002-11-11 Por tôpico Augusto César Morgado



Escolha um subconjunto com k elementos, o que voce pode fazer de C(n,k) modos.
O outro subconjunto deve ser um subconjunto do complementar: como o complementar
eh de tamanho 
n - k, este outro subconjunto pode ser escolhido de 2^(n-k) modos.
A resposta parece ser somatorio de C(n,k)*[2^(n-k)] com k variando de 0
a n. Essa soma vale (binmio de Newton) 3^n. 
Mas ha dois problemas: Primeiro, uma contagem dupla. Por exemplo, para o
conjunto {1, 2, 3, 4}, o par {1}, {2,3} eh contado duas vezes: uma quando
se escolhe o {1} como subconjunto e o {2,3} eh escolhido como o outro subconjunto
e vice-versa.
Segundo, o par vazio vazio soh eh contado uma vez.
A resposta eh 1 + a metade de (3^n - 1) = metade de (3^n + 1)
Bonito problema, Carlos Gomes!

cgmat wrote:
001701c28934$42e713a0$f270bfc8@41a7ime526d044j">
  
  
  Al pessoal, ser que algum poderia de
dar uma  dica na questo: 
  De quantas formas podemos
selecionar  dois subconjuntos disjuntos a partir de um conjunto finito com
n  elementos?
  Grato, C.Gomes.
  
  
  
  


Re: [obm-l] subconjuntos

2002-11-11 Por tôpico Augusto César Morgado



Os conjuntos sao disjuntos; nao sao necessariamente complementares.

Domingos Jr. wrote:
003501c2899f$79d20e90$2accfea9@gauss">
  
  seja N = 2k + s, com s = {0, 1}
  
  0) voc pode formar um subconjunto vazio
e outro  com 2k+s elementos
  1) um subconjunto com 1 elemento, outro
com  2k+s-1
  ...
  i) um com i elementos e outro com  2k+s-i
  
  se i  k estamos contando alguns subconjuntos
 duas vezes,
  logo pegamos i = k
  seja S a soma de i =0 at k de  Cn,i
  
  se N for mpar, temos
  2*S = 2^n = S = 2^(n-1)
  
  ex.
  {1, 2, 3}, S = 2^2 = 4
  {},{1, 2, 3} ; {
1},{2,3} ; {2},{1,3} ; {3},{1,2}
  
  se N for par, temos
  2*S + Cn,n/2 = 2^n
  =
  2*S - n!/[(n/2)!] = 2^n
  
  S = 2^(n-1) + 0,5Cn,n/2
  
  para N=4, temos S = 1 + 4 + 6 = 11 =2^3
+  0,5*C4,2
  
  
  

- Original Message - 

From:
brunolima


To:
[EMAIL PROTECTED]


Sent: Monday, November 11, 2002 10:44AM

Subject: Re: [obm-l] subconjuntos


Resposta bem grosseira: Divida o conjunto em dois subconjuntos um
   comk e outro com n-k elementos. Faca as combinaes('NescolhaK')e
multiplique por n, pois K varia de 1 a n.
cgmat [EMAIL PROTECTED]
 wrote:

  
  
  Al pessoal, ser que algum poderia
de dar uma  dica na questo: 
  De quantas formas
podemos  selecionar dois subconjuntos disjuntos a partir de um conjunto
finito  com n elementos?
  Grato, C.Gomes.
  
  
  
  Yahoo! GeoCities
  
Tudopara criar o seu site: ferramentas fceis de usar, espao de sobra
eacessrios.
  
  
  
  


Re: [obm-l] subconjuntos

2002-11-11 Por tôpico Augusto César Morgado



A resposta  a metade de (3^n +1).

Marcos Aurelio Almeida da Silva wrote:
[EMAIL PROTECTED]">
  pessoal desculpe mas essa resposta est errada, pois havero 3^n relaespossveis s que algumas delas so equivalentes...acho que d para ficar assim:como {(x,1),(y,2)...}  equivalente a {(x,2),(y,1),...}, logo para todarelao existe uma outra completamente equivalente  ela,fica1. 3^n-1: exclui o caso em que todos so relacionados ao zero, noformando conjuntos disjuntos.2. (3^n-1)/2: exclui todas as relaes equivalenteslogo N = (3^n-1)/2acho que dessa vez t tudo ok:. Marcos Aurlio Almeida da Silva.:..:. e-mail:   [EMAIL PROTECTED] .:..:. site  :   http://cin.ufpe.br/~maas .:.On Mon, 11 Nov
 2002, Marcos Aurelio Almeida da Silva wrote:
  
bom, imagine um conjunto:A = {a1, a2, ..., an}imagine a seguinte relao que acossia a cada elemento do conjunto A umvalor:R: A - {0,1,2}vamos formar os seguintes conjuntos:B = { x / (x,1) pertence a R}C = { x / (x,2) pertence a R}D = { x / (x,0) pertence a R}logo temos dois conjuntos disjuntos que so subconjuntos de A (B e C),e o conjunto D que  formado pelos elementos que no entram em nenhum dosoutros dois conjuntos. Para contar o nmero de  subconjuntos disjuntos s contar o nmero de relaes, pois a cada par de subconjuntoscorresponde uma relao e a cada releo corresponde um par de conjuntos,logo a resposta deve ser 3^n..:. Marcos Aurlio Almeida da Silva.:..:. e-mail:   [EMAIL PROTECTED] .:..:. site  :   http://cin.ufpe.br/~maas .:.On Mon, 11 Nov 2002, cgmat wrote:

  Al pessoal, ser que algum poderia de dar uma dica na questo:De quantas formas podemos selecionar dois subconjuntos disjuntos  a partir de um conjunto finito com n elementos?Grato, C.Gomes.
  
  
  
  =Instrues para entrar na lista, sair da lista e usar a lista emhttp://www.mat.puc-rio.br/~nicolau/olimp/obm-l.htmlO administrador desta lista  [EMAIL PROTECTED]=
  
  
  
  


[Fwd: Re: [obm-l] subconjuntos]

2002-11-11 Por tôpico Augusto César Morgado



Estou reenviando porque parece que ninguem leu.

 Original Message 

  

  From: 
  - Mon Nov 11 14:35:08 2002


  X-UIDL: 
  T=!!RO"![dc"!9g:"!


  X-Mozilla-Status: 
  0011


  X-Mozilla-Status2: 
  


  Return-Path: 
  [EMAIL PROTECTED]


  Received: 
  from sucuri.mat.puc-rio.br (sucuri.mat.puc-rio.br [139.82.27.7])	by
trex.centroin.com.br (8.12.5/8.12.1) with ESMTP id gABEfMfD009988	for [EMAIL PROTECTED];
Mon, 11 Nov 2002 12:41:22 -0200 (EDT)


  Received: 
  (from majordom@localhost)	by sucuri.mat.puc-rio.br (8.9.3/8.9.3)
id MAA19220	for obm-l-MTTP; Mon, 11 Nov 2002 12:34:29 -0200


  Received: 
  from trex-b.centroin.com.br (trex-b.centroin.com.br [200.225.63.136])	by
sucuri.mat.puc-rio.br (8.9.3/8.9.3) with ESMTP id MAA19216	for [EMAIL PROTECTED];
Mon, 11 Nov 2002 12:34:23 -0200


  Received: 
  from centroin.com.br (du105c.rjo.centroin.com.br [200.225.58.105])	(authenticated
bits=0)	by trex-b.centroin.com.br (8.12.5/8.12.1) with ESMTP id gABEY5Dh002564	for
[EMAIL PROTECTED]; Mon, 11 Nov 2002 12:34:10 -0200 (EDT)


  Message-ID: 
  [EMAIL PROTECTED]">[EMAIL PROTECTED]


  Date: 
  Mon, 11 Nov 2002 12:33:59 -0200


  From: 
  Augusto Csar Morgado [EMAIL PROTECTED]


  User-Agent: 
  Mozilla/5.0 (Windows; U; Win98; en-US; rv:0.9.4.1) Gecko/20020508
Netscape6/6.2.3


  X-Accept-Language: 
  en-us


  MIME-Version: 
  1.0


  To: 
  [EMAIL PROTECTED]


  Subject: 
  Re: [obm-l] subconjuntos


  References: 
  001701c28934$42e713a0$f270bfc8@41a7ime526d044j


  Content-Type: 
  multipart/alternative; boundary="050201010406050504070106"


  Sender: 
  [EMAIL PROTECTED]


  Precedence: 
  bulk


  Reply-To: 
  [EMAIL PROTECTED]


  X-UIDL: 
  T=!!RO"![dc"!9g:"!


  Status: 
  U

  



   Escolha um subconjunto com k elementos, o que voce pode fazer de C(n,k)
modos. O outro subconjunto deve ser um subconjunto do complementar: como
o complementar eh de tamanho 
 n - k, este outro subconjunto pode ser escolhido de 2^(n-k) modos.
 A resposta parece ser somatorio de C(n,k)*[2^(n-k)] com k variando de
0 a n. Essa soma vale (binmio de Newton) 3^n. 
 Mas ha dois problemas: Primeiro, uma contagem dupla. Por exemplo, para o 
conjunto {1, 2, 3, 4}, o par {1}, {2,3} eh contado duas vezes: uma quando 
se escolhe o {1} como subconjunto e o {2,3} eh escolhido como o outro subconjunto 
e vice-versa.
 Segundo, o par vazio vazio soh eh contado uma vez.
 A resposta eh 1 + a metade de (3^n - 1) = metade de (3^n + 1)
 Bonito problema, Carlos Gomes!

 cgmat wrote:
001701c28934$42e713a0$f270bfc8@41a7ime526d044j">
  
  
  Al pessoal, ser que algum poderia de 
dar uma  dica na questo: 
  De quantas formas podemos 
selecionar  dois subconjuntos disjuntos a partir de um conjunto finito com 
n  elementos?
  Grato, C.Gomes.
  
  
  
  
  


Re: [obm-l] Livros

2002-11-11 Por tôpico Augusto César Morgado



Mensagens pedindo informaoes sobre livros deveriam sempre vir acompanhadas
da cidade do remetente.
Eu ia responder exatamente como fez o Nicolau quando, por acaso, me dei conta
que o remetente era do Cear. Esses livros podem ser encontrados no Cear,
no h necessidade de encomend-los no Rio. Onde podem ser encontrados? Os
cearenses da lista (Paulo Rodrigues e outros) sabem, mas nao responderam
porque nao se deram conta que o remetente era cearense.
Morgado

Nicolau C. Saldanha wrote:
[EMAIL PROTECTED]">
  On Thu, Jan 01, 1998 at 06:04:47AM -0200, Fernando wrote:
  
Amigos Virtuais,Como poderia adquirir esses livros?a.. Olimpadas Brasileiras de Matemtica, 1a. a 8a. :Problemas e SoluesCompilado por lio Mega e Renate Watanabe.Sociedade Brasileira de Matemtica - SBM.a.. Olimpadas Brasileiras de Matemtica, 9a. a 15a.Luiz Amancio Machado de Sousa Jr.Edies UFC. Fortaleza - CE.a.. Olimpada de Matemtica do Estado do Rio de Janeiro (Problemas e Solues)Antonio Luiz Santos, Eduardo Wagner, Raul F. AgostinhoSociedade Brasileira de Matemtica - SBM.a.. Olimpadas de Matemtica 97 - Provas Compiladas e ResolvidasAntonio Caminha, Onofre Campos, Paulo Bonfim Gomes RodriguesEditora 7 de Setembro - Fortaleza - CE a.. Atenciosamente, a.. Fernando

Para os livros publicados pela SBM, tente www.sbm.org.brou escreva para a secretria da SBM encarregada de venda de livros.[]s, N.=Instrues para entrar na lista, sair da lista e usar a lista emhttp://www.mat.puc-rio.br/~nicolau/olimp/obm-l.htmlO administrador desta lista  [EMAIL PROTECTED]=






Re: [obm-l] teoria dos números

2002-11-05 Por tôpico Augusto César Morgado



Reenvio corrigindo um erro.

Augusto Csar Morgado wrote:
[EMAIL PROTECTED]">   
= 3 (mod 7)
 ^2 = 3^2 = 2 (mod 7)
 ^3 = 3*2 = 6 (mod7)
 ^4 = 2^2 = 4 (mod 7)
 ^5 = 5 (mod 7)
 ^6 = 1 (mod 7) 
 A partir daih, repete-se em ciclos de 6 (mais precisamente ^(a+6) =
^a)
 como  = 6*925 + 5, ^= ^5 = 5.
 Analogamente, ^ = 2 (mod 7) e ^+^=
7 = 0 (mod 7)
 Eder wrote:
  007e01c28435$d3a2f340$3c02fea9@Eder">


 
Gostaria de ajuda nestes problemas:

1)Se 2^k - 1,onde k  um inteiro maior 
que 2,  primo,prove que k  primo.
2)Mostre que ^() + ^() 
 divisvel  por 7.
3)Prove que se um dos nmeros 2^n -
1 e 2^n + 1   primo,ento utro  composto.










Re: [obm-l] Ajuda em probabilidade

2002-11-04 Por tôpico Augusto César Morgado



O problema  complicado, no sentido que exige um conhecimento especfico
de algumas tcnicas de probabilidade. Veja o livro do Feller (captulo 11),
na parte de Passeios Aleatrios e procure por Retorno  Origem.
A propsito, a resposta  1 - mdulo (2p-1)

Felipe Villela Dias wrote:
000a01c283a7$3e264df0$158c000a@computador">
  
  
  Um moeda  viciada, ou seja tem uma probabilidade p,
p  diferente de 50%, de dar cara e uma probabilidade 1 - p de dar coroa.
Sendo  assim, se voc jogar a moeda infinitas qual a probabilidade de que
em pelo menos  um instante o nmero de vezes que saiu cara vai ser igual
ao nmero de vezes que  saiu coroa?
  
  
  
  
---
Outgoing mail is certified Virus Free.
Checked by AVG  anti-virus system (http://www.grisoft.com
).
Version: 6.0.408 /  Virus Database: 230 - Release Date: 24/10/2002
  
  
  
  


Re: [obm-l] desafio !

2002-11-04 Por tôpico Augusto César Morgado


João Gilberto Ponciano Pereira wrote:
No seu contra-exenplo P(1) e P(5) nao tem homem comum.


Um comandante de companhia convocou voluntários para a constituição de 11
patrulhas. Todas elas são formadas pelo mesmo número de homens. Cada homem
participa de exatamente duas patrulhas. Cada duas patrulhas tem somente um
homem em comum. Determine o múmero de voluntários e integrantes de uma
patrulha. 

Pessoal

E se tivermos apenas 11 homens, com 2 homens por patrulha, de forma que
P(1) = H(1) e H(2)
...
P(N) = H(N) e H(N+1)
P(11) = H(11) e H(1)

SDS
JG

-Original Message-
From: Eder [mailto:edalbuquerque;uol.com.br]
Sent: Sunday, November 03, 2002 1:03 PM
To: [EMAIL PROTECTED]
Subject: Re: [obm-l] desafio !


Uma vez alguém me falou de uma analogia interessante que poderia ser
utilizada neste problema...
É o seguinte:

Consideremos um polígono convexo de 11 lados e,é claro,de 11 vértices.Você
posicionaria 1 homem em cada vértice,assim estes estariam em exatamente duas
patrulhas e cada duas patrulhas teriam um homem em comum.Agora, considere as
diagonais.Imaginando uma diagonal como um homem,vemos que aqui também é
respeitado o fato de que cada homem estaria em exatamente duas patrulhas e
cada duas patrulhas,ligadas pela diagonal,teriam um homem em comum.Assim o
total de homens é:

nº vértices+nº diagonais= 11 +11(11-3)/2 = 55.


Seja n o número de homens por patrulha.Temos que 11*n=2*55 (pois cada homem
foi contado duas vezes),daí n=10.

Espero que esteja certo.


Eder


- Original Message - 
From: Wander  mailto:wander29br;hotmail.com Junior 
To: [EMAIL PROTECTED] mailto:obm-l;mat.puc-rio.br  
Sent: Sunday, November 03, 2002 2:09 PM
Subject: [obm-l] desafio !

Dúvida:

Um comandante de companhia convocou voluntários para a constituição de 11
patrulhas. Todas elas são formadas pelo mesmo número de homens. Cada homem
participa de exatamente duas patrulhas. Cada duas patrulhas tem somente um
homem em comum. Determine o múmero de voluntários e integrantes de uma
patrulha.


Agradeço desde já.
Wander

=
Instruções para entrar na lista, sair da lista e usar a lista em
http://www.mat.puc-rio.br/~nicolau/olimp/obm-l.html
O administrador desta lista é [EMAIL PROTECTED]
=




=
Instruções para entrar na lista, sair da lista e usar a lista em
http://www.mat.puc-rio.br/~nicolau/olimp/obm-l.html
O administrador desta lista é [EMAIL PROTECTED]
=



Re: [obm-l] teoria dos números

2002-11-04 Por tôpico Augusto César Morgado



 = 3 (mod 7)
^2 = 3^2 = 2 (mod 7)
^3 = 3*2 = 6 (mod7)
^4 = 2^2 = 4 (mod 7)
^5 = 5 (mod 7)
^6 = 1 (mod 7) 
A partir daih, repete-se em ciclos de 6 (mais precisamente ^(a+6) = ^a)
como  = 6*925 + 5, ^= ^5 = 5.
Analogamente, ^ = 4 (mod 7) e ^+^=9 = 2 (mod 7)
Eder wrote:
007e01c28435$d3a2f340$3c02fea9@Eder">
  
  
   
  Gostaria de ajuda nestes problemas:
  
  1)Se 2^k - 1,onde k  um inteiro maior
que 2,  primo,prove que k  primo.
  2)Mostre que ^() + ^()
 divisvel  por 7.
  3)Prove que se um dos nmeros 2^n - 1
e 2^n + 1   primo,ento utro  composto.
  
  
  
  
  
  


Re: [obm-l] Base 7

2002-11-04 Por tôpico Augusto César Morgado
6*7*7

Igor GomeZZ wrote:





[IME1992] Calcule quantos números naturais de 3 algarismos existem no
sistema de base 7.



Infelizmente, não possuo a resposta... Qualquer dica, blz!

Fui!


### Igor GomeZZ 
UIN: 29249895
Vitória, Espírito Santo, Brasil
Criação: 4/11/2002 (19:56)

Pare para pensar:

Nunca desencoraje ninguém que
continuamente faz progresso, não
importa quão devagar. (Platão)




=
Instruções para entrar na lista, sair da lista e usar a lista em
http://www.mat.puc-rio.br/~nicolau/olimp/obm-l.html
O administrador desta lista é [EMAIL PROTECTED]
=





=
Instruções para entrar na lista, sair da lista e usar a lista em
http://www.mat.puc-rio.br/~nicolau/olimp/obm-l.html
O administrador desta lista é [EMAIL PROTECTED]
=



Re: [obm-l] duvida conceitual sobre probabilidades

2002-11-04 Por tôpico Augusto César Morgado
Voce pode fazer C(3,1)*C(4,1)*C(2,1)/C(9,3), que da a mesma coisa, mas 
sempre dah a mesma coisa tirar sucessivamente sem reposiçao ou 
simultaneamente porque os casos favoraveis e os possiveis ficam 
multiplicados pelo mesmo fator.

niski wrote:

Ola pessoal..o problema é o seguinte..
Considere um balaio onde se encontram 3 bolas brancas, 4 bolas 
vermelhas e 2 bolas pretas.
Qual é a probabilidade de se tirar simultaneamente 3 bolas de cores 
diferentes?

Bom, o inicio do problema me parece facil:

Por exemplo que a primeira bola seja branca a segunda seja verm e a 
terceira seja preta, a probabilidade pedida é
B   V  P
3/9 * 4/8 * 2/7
Ai que entra a minha duvida...vi que o meu professor multiplicou esse 
resultado por 3!, já que pode-se tirar as 3 bolas nao necessariamente 
nessa ordem (B,V,P)...bom eu aceito isso no caso de se ir tirando as 
bolas do balaio uma por uma ...mas como pode-se fazer distincao, por 
exemplo, entre tirar primeiro B, depois V e dpois P e tirar primeiro P 
depois V e depois B se estou tirando as bolas simultaneamente? ou 
seja..eu enfio a mao no balaio e tiro as 3 bolas...nao seria o mesmo 
evento pra minha mao encaixar 3 bolas diferentes sejam elas quaisquer 
3 bolas diferentes?
espero que tenha sido claro!
obrigado

=
Instruções para entrar na lista, sair da lista e usar a lista em
http://www.mat.puc-rio.br/~nicolau/olimp/obm-l.html
O administrador desta lista é [EMAIL PROTECTED]
=




=
Instruções para entrar na lista, sair da lista e usar a lista em
http://www.mat.puc-rio.br/~nicolau/olimp/obm-l.html
O administrador desta lista é [EMAIL PROTECTED]
=



Re: [obm-l] Re:[obm-l] desafio(correção)

2002-11-04 Por tôpico Augusto César Morgado



Esse seu argumento eh perigoso. Considere um segmento AB de comprimento x.
Para ir de A a B, anda-se x. Pense agora num triangulo equilatero ABC. Para
ir de A a B via C, anda-se 2x.
Agora quebre AB ao meio, no ponto M. Para ir de A a B em linha reta via M,
anda-se x. Faa a mesma coisa do triangulo equilatero sobre AM e sobre MB,
anda-se 2x. Quebre AM e MB ao meio, etc. No limite, na reta voce anda x e
no zigue-zague, 2x. So que no limite, a reta e o zigue-zague se confundem.
Logo, x = 2x e 1=2.

glauber.morais wrote:
H52HKA$[EMAIL PROTECTED]">
  

  Ol,   Algum seria capaz de provar o seguinte lim sem utilizar o lim fundamental do sen:   lim x.tg(n/x)=n   x-inf  ou  lim x.sen(n/x)=n  x-inf  oi..
  
  Considera-se uma circunferncia de centro "A" e raio "R". E um tringulo retngulo "ABC", sendo os cateto AB=R e BC,  "a"  o ngulo CB. Para "a" infinitamene pequeno, o cateto BC se confunde com a circunferncia.Iguala-se ,ento, o semi-permetro da circunferncia, calculado atravs do raio da circunferncia e atravs d
  
  o 
  
somatrio de vrios "CB"s dispostos lado a lado com "A"

 

  no centro da circunferncia. A partir da, deduz-se o lim. proposto.   Desculpem qualquer confuso causada pela falta de recurso do teclado...   
  
_(ver correo na questo)


__

  ___
  
Encontre sempre uma linha desocupada com o Discador B


OL

  !
  
http://sac.bol.com.br/discador.htmlAinda no tem AcessoBOL? Assine j! http://sac.bol.co


m.

  br
  
=


==

  ==
  
Instrues para entrar na lista, sair da lista e usar


 a

   lista em
  
http://www.mat.puc-rio.br/~nicolau/olimp/obm-l.htmlO administrador desta lista  [EMAIL PROTECTED]-


rio.br

  
=


==

  ==
   ___
  
  ___
  
Encontre sempre uma linha desocupada com o Discador BOL

!

  http://sac.bol.com.br/discador.htmlAinda no tem AcessoBOL? Assine j! http://sac.bol.com.
  
  br
  
===

==

  Instrues para entrar na lista, sair da lista e usar a
  
   lista em
  
http://www.mat.puc-rio.br/~nicolau/olimp/obm-l.htmlO administrador desta lista  [EMAIL PROTECTED]===

==
 __Encontre sempre uma linha desocupada com o Discador BOL!http://sac.bol.com.br/discador.htmlAinda no tem AcessoBOL? Assine j! http://sac.bol.com.br=Instrues para entrar na 

Re: [obm-l] ... ajuda ...

2002-11-02 Por tôpico Augusto César Morgado



Nao seria geometrica ou harmonica em vez de ponderada?

Eleu Natalli wrote:
[EMAIL PROTECTED]">
  alguem pode demonstar para mim que a media
aritmetica de "n" numeros  sempre maior ou igual que a mdia ponderada dos
mesmos "n" numeros ?
  obs: ultilizando numeros positivos ...
  
  
  Yahoo! GeoCities
  
 Tudo para criar o seu site: ferramentas fceis de usar, espao de sobra
e acessrios. 
  
  
  


Re: [obm-l] binômio de newton

2002-11-01 Por tôpico Augusto César Morgado
Não, só se a=1.

pichurin wrote:


(a + x)^n
x é um número bem pequen0(entre zero e um)
Ex: (1 + 0,05)^32

Como calcular isso pelo Binômio de Newton(calcular o
valor aproximado)

essa aproximação pode ser dada por a + nx?

___
Yahoo! GeoCities
Tudo para criar o seu site: ferramentas fáceis de usar, espaço de sobra e acessórios.
http://br.geocities.yahoo.com/
=
Instruções para entrar na lista, sair da lista e usar a lista em
http://www.mat.puc-rio.br/~nicolau/olimp/obm-l.html
O administrador desta lista é [EMAIL PROTECTED]
=





=
Instruções para entrar na lista, sair da lista e usar a lista em
http://www.mat.puc-rio.br/~nicolau/olimp/obm-l.html
O administrador desta lista é [EMAIL PROTECTED]
=



Re: [obm-l] obm

2002-10-31 Por tôpico Augusto César Morgado



www.obm.org.br

Margarida Lanna wrote:
009001c28073$02bf96e0$4326fbc8@abc">
  
  
  Gostaria de receber questes antigas de
olimpadas  de matemtica com gabarito.
  
  obrigada,
  
  Margarida Lanna
  
  
  
  


Re: [obm-l] AFINAL-QUEM ESTA CERTO?????

2002-10-31 Por tôpico Augusto César Morgado





bruno lima wrote:
[EMAIL PROTECTED]">
  Qual soluo est correta  a minha ou essa??Acho que a correta  a seguinte: 610 = 2*305 = 2(304 + 1) = 2(2*152 + 1 ) = 2 + 152*2^2= 2 + 19*2^5 = 2 +(16+3)*2^5 = 2 + 2^5 + 2^6 + 2^9 Ento o cara acertou 4 perguntas: a segunda, a sexta,a stima e a dcima. 
  
  
Est certa. Morgado
  [EMAIL PROTECTED]">
 Marcelo Souza[EMAIL PROTECTED] wrote: Bom, acho que do jeito que eu fiz eh mais ou menos amesma coisa, veja:x_1+2x_2+4x_3+...+512x_10=610onde x_i pertence a {0,1}Ele deve ter acertado a pergunta 10, caso contrrio,'no daria para obter tal pontuacao.

At aqui, timo.
A partir daqui, na hora do analogamente  fcil concluir que... houve uma
distraaozinha!
[EMAIL PROTECTED]">
  ..observando tal fato, eh fcil concluirque ele acertou as perguntas 9,8,5,2...zerando asoutras.Falow[]'s MarceloFrom: Gabriel Prgola Reply-To: [EMAIL PROTECTED]To: "Obm-l" Subject: [obm-l] Problema doMrcio - jogo de tvDate: Sun, 13 Oct 2002 20:06:29-0300
  
E a pessoal,Estava olhando o problema que o Mrcio mandou para a

lista:Em um jogo de televiso, um candidato deve responder a10 perguntas. A primeira vale 1 ponto, a segunda vale2 pontos, e assim, sucessivamente, dobrando sempre. Ocandidato responde a todas as perguntas e ganha ospontos correspondentes s respostas que acertou, mesmoque erre algumas. Se o candidato obteve 610 pontos,quantas perguntas acertou?E vi a soluo usando nmero binrios (colocando nabase dois)..Gostaria de saber se existe alguma outra forma deresolver este problema, ese sim, como?Abrao,Gabriel___Yahoo! GeoCitiesTudo para criar o seu site: fe
rramentas fceis de usar, espao de sobra e acessrios.http://br.geocities.yahoo.com/=Instrues para entrar na lista, sair da lista e usar a lista emhttp://www.mat.puc-rio.br/~nicolau/olimp/obm-l.htmlO administrador desta lista  [EMAIL PROTECTED]=






Re: [obm-l] tradução de manifold

2002-10-27 Por tôpico Augusto César Morgado



Variedade

Jose Francisco Guimaraes Costa wrote:
005801c27d72$9a11d980$9c02dcc8@jf">
  
  
  Em matemtica, qual a traduo para portugus
da  palavra "manifold"?
  
  JF
  
  
  
  


Re: [obm-l] Agrupamento

2002-10-22 Por tôpico Augusto César Morgado
a) 2 operarios e 3 empresarios: C(10,2).C(5,3) = 45*10 = 450
b) 3 operarios e 2 empresarios: C(10,3).C(5,2) = 120*10 = 1200
Resposta: 1650
Gabriel Pérgola wrote:


Gostaria de ver a resolução deste problema:

Pretende-se formar uma comissão de 5 membros a partir de um grupo de 10
operários e 5 empresários, de modo que nessa comissão haja pelo menos dois
representantes de cada uma das classes. O total de diferentes comissões que
podem ser assim, formadas é?

a) 1000
b) 185
c) 19400
d) 1750
e) 1650


[]'s
Gabriel

=
Instruções para entrar na lista, sair da lista e usar a lista em
http://www.mat.puc-rio.br/~nicolau/olimp/obm-l.html
O administrador desta lista é [EMAIL PROTECTED]
=





=
Instruções para entrar na lista, sair da lista e usar a lista em
http://www.mat.puc-rio.br/~nicolau/olimp/obm-l.html
O administrador desta lista é [EMAIL PROTECTED]
=



Re: [obm-l] DÚVIDA

2002-10-20 Por tôpico Augusto César Morgado





Wagner wrote:
f(x) = (b^x + c^x)^(1/x) = exp [(1/x)
ln (b^x + c^x)]
f'(x) = exp [(1/x) ln (b^x + c^x)] * 
{(- 1/x^2) ln (b^x + c^x) + (1/x) [(b^x * lnb + c^x * ln c)/ (b^x + c^x)]

005d01c2785e$33c67be0$03909ec8@u2z7z2">
  Oi para todos!
  
   Estava resolvendo um problema e  me
deparei com isso:
  
  Se f(x) = (b^x + c^x)^(1/x), quanto vale
f '(x) em  funo de b e c?
  
  A quem conseguir me ajudar agradeo
  
  Andr T.
  
  
  
  


Re: [obm-l] Uma ajudinha por favor...

2002-10-20 Por tôpico Augusto César Morgado
Eh facil ver que nao ha raiz negativa (se x for negativo, as quatro 
parcelas serao negativas e a soma dara negativa e nao zero).
Tampouco ha raiz positiva menor que 1 (x^3 - x^2 = x^2 (x - 1) sera 
negativo; a soma das quatro parcelas sera menor que - 1).
Logo, voce tem que procurar raiz maior que 1.
Com uma calculadora ou um aplicativo de computador voce achara  1,3532

filipe falcão wrote:

Olá pessoal,

tava tentando resolver mas não saiu... x³-x²+x-2=0 . É isso ai, se 
alguem puder ajudar eu agradeço desde já.

Filipe Falcão






_
MSN Hotmail, o maior webmail do Brasil. http://www.hotmail.com

=
Instruções para entrar na lista, sair da lista e usar a lista em
http://www.mat.puc-rio.br/~nicolau/olimp/obm-l.html
O administrador desta lista é [EMAIL PROTECTED]
=




=
Instruções para entrar na lista, sair da lista e usar a lista em
http://www.mat.puc-rio.br/~nicolau/olimp/obm-l.html
O administrador desta lista é [EMAIL PROTECTED]
=



Re: [obm-l] log natural =log neperiano ?

2002-10-19 Por tôpico Augusto César Morgado
Bobagem! O que o mundo todo chama de logaritmo neperiano eh a mesma 
coisa que logaritmo natural. Agora, o que o mundo todo chama de 
logaritmo neperiano nao eh exatamente a mesma coisa que foi inventada 
por Neper e sim o resultado de uma pequena modificaçao na criaçao do 
Neper. Naquele tempo, sem calculadoras e computadores, qualquer coisinha 
que facilitasse contas era ouro puro. O logaritmo inventado por Neper, 
por exemplo, entre outras pequenas modificaçoes, eh o nosso multiplicado 
por uma certa potencia de 10 (10 elevado a 9, creio) para evitar virgula 
nos calculos etc. Se voce quiser saber exatamente o que Neper fez, va a 
um livro de Historia da Matematica.
Mas todo mundo chama de neperiano o logaritmo natural, que eh o 
resultado de pequenas modificaçoes na invençao do Neper.
Morgado

adr.scr.m wrote:

vi num livro uma vez dizendo que logaritmo 
natural(base e) eh diferente do logaritmo 
neperiano, e gostaria de saber se eh 
verdade,porque todos os professores que eu 
conheco (pessoalmente)os tratam como iguais.
obrigado.
[]'s.
Adriano.


__
BOL - três anos com você. Venha pra festa e ganhe uma viagem!
http://especial.bol.com.br/2002/3anos
Ainda não tem AcessoBOL? Assine já! http://sac.bol.com.br


=
Instruções para entrar na lista, sair da lista e usar a lista em
http://www.mat.puc-rio.br/~nicolau/olimp/obm-l.html
O administrador desta lista é [EMAIL PROTECTED]
=




=
Instruções para entrar na lista, sair da lista e usar a lista em
http://www.mat.puc-rio.br/~nicolau/olimp/obm-l.html
O administrador desta lista é [EMAIL PROTECTED]
=



Re: [obm-l] Benedito de Moraes

2002-10-11 Por tôpico Augusto César Morgado



Prezado Marcos,
Instrues para a compra de livros da SBM encontram-se em
www.sbm.org.br 
Um abrao
Morgado

Marcos Gomes de Melo wrote:
004e01c27175$c02d45e0$2b69ccc8@terra">
  Caro Morgado,Agradeo sua resposta. Se voc souber onde posso adquirir o livro do Elon,que voc citou, fico-lhe agradecido.Fui aluno da ltima turma do velho "Biu" em 1960 e 1961, quando ele parou deensinar Matemtica. Eram dois anos, onde no primeiro dava a matria de todoo ginsio (quinta a oitava) e no segundo a de todo o cientfico. Era como sefosse um cursinho s de Matemtica para preparar para o vestibular.O Professor Benedito era autodidata, no tinha nem o curso primrio, e erafeirante, mas transformou-se em excelente Professor ao desenvolver nosalunos o gosto pela Matemtica, conseguindo desmistific-la. No raro alunosmedocres, em Matemtica, aps estudar com ele se transformavam em bonsalunos na matria. Foi o meu caso, que ia fazer medicina por temer aMatemti
ca. Graas a ele mais tarde consegui enfrentar o vestibular do ITA eda POLI,  passar em ambos e formar-me com "Meno Honrosa" em Matemticapelo ITA..Rendo aqui minhas homenagens ao velho mestre.SDS,Marcos Melo.- Original Message -From: "Augusto Csar Morgado" [EMAIL PROTECTED]To: [EMAIL PROTECTED]Sent: Wednesday, October 09, 2002 11:42 AMSubject: Re: [obm-l] Benedito de Moraes
  
O professor Benedito foi professor de dois ilusres matematicosbrasileiros, Elon Lages Lima e Manfredo Perdigao do Carmo.Elon publicou, na Coleao do Professor de Matematica da SBM, um livroque reune artigos por ele escritos e que se chama Meu Professor deMatematica. O titulo do livro eh o titulo do primeiro artigo e se refereao professor Benedito.MorgadoMarcos Melo wrote:

  Algum da lista foi aluno, conheceu ou j ouviu falar do Prof.Benedito de Moraes, de Alagoas?SDS,Marcos Melo.=Instrues para entrar na lista, sair da lista e usar a lista emhttp://www.mat.puc-rio.br/~nicolau/olimp/obm-l.htmlO administrador desta lista  [EMAIL PROTECTED]=
  
  =Instrues para entrar na lista, sair da lista e usar a lista emhttp://www.mat.puc-rio.br/~nicolau/olimp/obm-l.htmlO administrador desta lista  [EMAIL PROTECTED]=Esta mensagem foi verificada pelo E-mail Protegido Terra.Scan engine: VirusScan / Atualizado em 02/10/2002 / Verso: 1.3.13Proteja o seu e-mail Terra: http://www.emailprotegido.terra.com.br/
  
  =Instrues para entrar na lista, sair da lista e usar a lista emhttp://www.mat.puc-rio.br/~nicolau/olimp/obm-l.htmlO administrador desta lista  [EMAIL PROTECTED]=
  
  
  
  


Re: [obm-l] Re:

2002-10-07 Por tôpico Augusto César Morgado





diegoalonsoteixeira wrote:
H3METQ$[EMAIL PROTECTED]">
  8x^3+kx^2-18x+9   as raizes so a,-a,e p(a outra raiz)Soma das raizes=-k/8=a-a+p      p=k/8 AQUI HA UM ERRINHO DE SINALproduto dois a dois das raizes=ak/8-a^2-ak/8=-a^2=-18/8   .  a=+-raiz de 18/8 produto das tres raizes=-9/8=a(-a)k/8=-a^2k/8=-18k/64=-9/8k=9*8/18=4espero no ter errado __Encontre sempre uma linha desocupada com o Discador BOL!http://www.bol.com.br/discadorAinda no tem AcessoBOL? Assine j! http://www.bol.com.br/acessobol
  Quem pode auxiliar a resolver:Se a equao 8x + kx - 18x + 9 = 0 tem razes reais "a" e "-a", ento o valor de k :(a) 9/4  (b) 2 (c) 9/8(d) -2(e)-4Obrigado,Guilherme_Converse com seus amigos online, faa o download grtis do MSN Messenger: http://messenger.msn.com.br=Instrues para entrar na lista, sair da lista e usar a lista emhttp://www.mat.puc-rio.br/~nicolau/olimp/obm-l.htmlO administrador desta lista  [EMAIL PROTECTED]=
  
  
  
  


[obm-l] CUIDADO COM MENSAGEM

2002-10-02 Por tôpico Augusto César Morgado

Aos que nao estao com antivirus atualizado:
Nao abram o anexo de uma mensagem sobre JOGOS supostamente enviada pelo 
Paulo Rodrigues (pauloemanu).
Esta infectado.
Morgado

=
Instruções para entrar na lista, sair da lista e usar a lista em
http://www.mat.puc-rio.br/~nicolau/olimp/obm-l.html
O administrador desta lista é [EMAIL PROTECTED]
=



Re: [obm-l] analítica

2002-09-27 Por tôpico Augusto César Morgado



Ligue a origem ao centro. Os pontos de interseao com a circunferencia sao
os que estao a maior e a menor distancia da origem

Eder wrote:
003b01c2667b$9386f0e0$d5d9fea9@Eder">
  
  
  Ol,
  
  Dada a equao de uma circunferncia :
 (x-4)+(y-3)=9,como fao para achar o ponto sobre a mesma que est a menor
 distncia da origem?Algum poderia dar uma dica?
  
  
  
  
  
  
  
  
  


Re: [obm-l] combinatoria

2002-09-26 Por tôpico Augusto César Morgado



Pisei na bola no exercicio 2. Nao reparei que era de 1 a 9.
Corrigindo, 
Podemos escolher 3 impares de C(5,3)=10 modos e 2 pares de C(4,2) = 6 modos. Escolhidos os algarismos, ha 5!=120 modos de arruma-los, e a resposta seria 10 x6 x 120 = 7 200. 


Augusto Cesar de Oliveira Morgado wrote:
[EMAIL PROTECTED]">
  Em Wed, 25 Sep 2002 18:17:17 -0300, Carlos Roberto de Moraes [EMAIL PROTECTED] disse:
  
Podem me ajudar?1)Um tabuleiro quadrado dispe de 9 orificios dispostos em 3 linhas e 3 colunas. De quantas maneiras podemos colocar 3 bolas de modo que os orificios ocupados no fiquem alinhados? Diagonais so consideradas tipos de alinhamento.2) O total de nmeros constituidos de 3 algarismos impares e 2 algarismos pares que podem ser formados com os algarismos de 1 a 9, sem repetio  igual a qto?

1) Supondo as bolas iguais, ha C(9,3)= 84 modos de coloca-las no tabuleiro. Excluindo as 3 horizontais, as 3 verticais e as 2 diagonais, obtemos a resposta 84-8 = 76.Supondo as bolas diferentes, a resposta passa a ser 76 x 3!2)Podemos escolher 3 impares de C(5,3)=10 modos e 2 pares de C(5,2) = 10 modos. Escolhidos os algarismos, ha 5!=120 modos de arruma-los, e a resposta seria 10 x 10 x 120 = 12 000. Entretanto, devemos excluir os numeros comeados por zero que sao4(numero de modos de escolher o outro algarismo par) x 10 (numero de modos de escolher os algarismos impares) x 4!(numero de modos de arruma-los com o zero no primeiro lugar).A resposta eh  12 000 - 960 = 11 040.=Instrues para entrar na lista, sair da lista e usar a lista emh
ttp://www.mat.puc-rio.br/~nicolau/olimp/obm-l.htmlO administrador desta lista  [EMAIL PROTECTED]=






Re: [obm-l] numero de dígitos

2002-09-23 Por tôpico Augusto César Morgado



Que eh soma pelo produto?

diegoalonsoteixeira wrote:
H2VA3A$[EMAIL PROTECTED]">
  Recebi uma questo que no consegui fazer:quanto vale a soma pelo produto dos seis primeiros digitos de (7^4600!*7^460!)/(7^10!*7^46!)No estou acostumado a esse tipo de questo, quem responde-la por favor faa detalhadamente.Obrigado __Encontre sempre uma linha desocupada com o Discador BOL!http://www.bol.com.br/discadorAinda no tem AcessoBOL? Assine j! http://www.bol.com.br/acessobol
  From: Wagner
  
Oi pessoal!Li em uma reportagem que um tal de nmero gugol  10^100 e que outro nmero

chamado de gugolplex  igual  gugol^gugol. Fiquei pensando, o que seriamaior, (1gugol)! ou 1 gugolplex. Como acho a resposta disso?

  Andr T.
  
  O que  maior: 1 * 2 * 3 * 4 * ... * n ou n * n * n * n * ... * n ( nvezes )?=Instrues para entrar na lista, sair da lista e usar a lista emhttp://www.mat.puc-rio.br/~nicolau/olimp/obm-l.htmlO administrador desta lista  [EMAIL PROTECTED]=
  
  
  
  


Re: [obm-l] Será que ninguém me ajuda???

2002-09-19 Por tôpico Augusto César Morgado



   Contando o dia do nascimento como dia 0, os dias de coincidncia so
da forma 6+23r = 7+29s = 8+33t.
 Temos umas diofantinas para resolver.
 A primeira delas 
 23r-29s=1
 Resolvendo-a obtemos
 r = -5 +29k
 s = 4 + 23k, k inteiro.
 Substituindo, 23*29k - 33t = 117
 Resolvendo-a,
 k = 33n +12
 t = 667n + 239.
 Portanto as solues inteiras do sistema l de cima so
 r = 957n + 343
 s = 759n + 280
 t = 667n + 239

 Em particular (s calculei isso tudo para que possam conferir e detectar 
eventuais erros de conta) os ciclos coincidem nos dias da forma 6 + 23r = 
22 011 * n + 7 895
 A primeira coincidncia se d no dia 7 895 (cerca de 22 anos aps o nascimento).






 e isso mesmo wrote:
[EMAIL PROTECTED]">
  
  Companheiros continuo esperando ajuda de algum
  
  
  
 "A teoria do Biorritmo diz que os estados fsico, mental e emocional de
uma pessoa oscilam periodicamente, a partir do dia do nascimento, em ciclos
de 23 dias, 29 dias e 33 dias, respectivamente. Dado que os dias mais positivos 
dos ciclos fsico, mental e emocional so, respectivamente, o sexto, o stimo 
e o oitavo de cada ciclo, nos primeiros dez anos de vida de uma pessoa, quantas 
vezes os trs ciclos esto simultaneamente no ponto mximo?"
  
 
  
 Tchau!
  
 
  
  
  Aproveite melhor a Web. Faa o download GRTIS do MSN Explorer : 
 http://explorer.msn.com.br/intl.asp#po
  
  
  
  
  
  


[obm-l] [Fwd: RE: Poliômios]

2002-09-19 Por tôpico Augusto César Morgado



Este problema foi proposto (novamente) ha pouco tempo. Estou reenviando-o
para a lista como um auxilio para quem o propos recentemente.
Morgado

 Original Message 

  

  From: 
  - Tue Oct 16 21:20:31 2001


  X-UIDL: 
  ~!#!EE:!!%EP!!@Z5"!


  X-Mozilla-Status: 
  0001


  X-Mozilla-Status2: 
  


  Return-Path: 
  [EMAIL PROTECTED]


  Received: 
  from triceratops.centroin.com.br (mail-gw2.centroin.com.br [200.225.50.252])	by
trex.centroin.com.br (8.10.1/8.10.1) with ESMTP id f9GDLje03205	for [EMAIL PROTECTED];
Tue, 16 Oct 2001 11:21:45 -0200 (EDT)


  Received: 
  from matinta.mat.puc-rio.br (matinta.mat.puc-rio.br [139.82.27.1])	by
triceratops.centroin.com.br (8.10.1/8.10.1) with ESMTP id f9GDLZR29426	for
[EMAIL PROTECTED]; Tue, 16 Oct 2001 11:21:37 -0200 (EDT)


  Received: 
  (from majordom@localhost)	by matinta.mat.puc-rio.br (8.9.3/8.9.3)
id LAA15406	for obm-l-list; Tue, 16 Oct 2001 11:01:20 -0200


  Received: 
  from pegasus.prt15.gov.br ([200.245.30.130])	by matinta.mat.puc-rio.br
(8.9.3/8.9.3) with ESMTP id LAA15399	for [EMAIL PROTECTED]; Tue,
16 Oct 2001 11:01:03 -0200


  Received: 
  (from daemon@localhost)	by pegasus.prt15.gov.br (8.11.1/8.11.1)
id f9GDBvW62656	for [EMAIL PROTECTED]; Tue, 16 Oct 2001 11:11:57
-0200 (BRST)	(envelope-from [EMAIL PROTECTED])


  Received: 
  from codinsec.prt15.gov.br (CODINSEC.prt15.gov.br [192.168.1.63])	by
pegasus.prt15.gov.br (8.11.1/8.11.1) with SMTP id f9GDBvg62651	for [EMAIL PROTECTED];
Tue, 16 Oct 2001 11:11:57 -0200 (BRST)	(envelope-from [EMAIL PROTECTED])


  Received: 
  by codinsec.prt15.gov.br with Microsoft Mail	id [EMAIL PROTECTED]">[EMAIL PROTECTED];
Tue, 16 Oct 2001 11:11:18 -0300


  Message-ID: 
  [EMAIL PROTECTED]">[EMAIL PROTECTED]


  From: 
  Eduardo Grasser [EMAIL PROTECTED]


  To: 
  "'[EMAIL PROTECTED]'" [EMAIL PROTECTED]


  Subject: 
  RE: Polimios


  Date: 
  Tue, 16 Oct 2001 11:10:54 -0300


  MIME-Version: 
  1.0


  Content-Type: 
  multipart/mixed; boundary=" =_NextPart_000_01C15633.49ABC800"


  X-Sanitizer: 
  Este EMail foi desinfectado!


  X-Sanitizer-URL: 
  http://www.prt15.gov.br/


  X-Sanitizer-Rev: 
  $Id: sanitizer.pl,v 1.35 2001/02/01 00:10:46 bre Exp $


  Sender: 
  [EMAIL PROTECTED]


  Precedence: 
  bulk


  Reply-To: 
  [EMAIL PROTECTED]


  X-UIDL: 
  ~!#!EE:!!%EP!!@Z5"!


  Status: 
  U

  



 a pressa... a idia  essa mesma. O resto  sentar e desenvolver a idia...
Quase nunca consigo respostas corretas se no pego uma folha de papel e rescunho um pouco :-D

obrigado

Eduardo

--
De:	Eric Campos Bastos Guedes[SMTP:[EMAIL PROTECTED]]
Enviada em:	Segunda-feira, 15 de Outubro de 2001 19:01
Para:	[EMAIL PROTECTED]
Assunto:	RES: Polimios

Eu nao faria melhor...
Mas tem uns erros nas contas que nao invalidam a solucao.
O resto certo eh

-2x^3-2x^2+x+5

Eric.


-MENSAGEM ORIGINAL ABAIXO

p(x) = q(x)(x^4 + x^2 + 1) + ax^3 + bx^2 + cx + d
(quis com isso dizer que o resto  um polinmio de grau 3)

Divido por x^2 + x + 1, e tenho que a primeira parte d zero pois x^2 + x +
1 divide x^4 + x^2 + 1 e a segunda d r
esto (a-c)x + d-b+a = 3x + 5

Divido por x^2 - x + 1, e tenho que a primeira parte d zero pois x^2 - x +
1 divide x^4 + x^2 + 1 e a segunda d resto (c-2a-b)x + d-a-b = -x + 9

Assim,  s resolver o sistema
a-c = 3
a-b+d = 5
-2a-b+c = -1
-a-b+d = 9

e achar o polinmio -2x^3 - 5x + 7 como resto

Acho que  isso salvo erros de conta, j que fiz correndo.

Eduardo Grasser
Campinas sp

--
De:	Ren Retz[SMTP:[EMAIL PROTECTED]]
Enviada em:	Segunda-feira, 15 de Outubro de 2001 15:54
Para:	[EMAIL PROTECTED]
Assunto:	Polimios

Sabe-se que os restos da diviso de um polinmio p(x) por x^2 + x + 1 e
x^2 - x + 1 so repsctivamente  3x + 5  e  -x + 9. Determine o resto de p(x)
por x^4 + x^2 + 1.












nsmail.tmp
Description: Binary data


Re: [obm-l] polinomios

2002-09-18 Por tôpico Augusto César Morgado



1) Dados: f(x) = (x+2) Q(x)
f(x) = (x^2+4) P(x) + (x+1)
Queremos
f(x) = (x+2)(x^2+4) S(x) + (Ax^2+Bx+C)
Para calcular A, B e C, faa x igual a -2, 2i e -2i.
Obtem-se o sistema
f(-2) = 4A -2B +C
f(2i) = -4A +2Bi +C
f(-2i) = -4A -2Bi + C
Os dados mostram que f(-2) = 0, f(2i) = 1+2i e f(-2i) = 1-2i.
Resolvendo o sistema, B=1, C=3/2, A=1/8.
O resto eh (1/8)(x^2) + (3/2)x + 1.
2)Observe que  x^4+x^2+1 = (  x^2+x+1)(
x^2-x+1)
Proceda analogamente. Os valores inteligentes para x sao as raizes de 
  x^2+x+1 e x^2-x+1.


Carlos Roberto de Moraes wrote:
000801c25e89$6f1d0ae0$[EMAIL PROTECTED]">
  
  
  Alguem pode me ajudar com esses 2 exerccios?
  
  1) Um polinomio f, dividido por x+2 e x^2+4 d restos
0 e x+1,  respectivamente. Qual  o resto da diviso de f por (x+2)(x^2+4)?
  
  2) Sabe-se que os restos da diviso de um polinomio
p(x) por  x^2+x+1 e x^2-x+1 so, respectivamente, 3x+5 e -x+9. Determine
o resto da  diviso de p(x) por x^4+x^2+1.
  
  
  
  


Re: [obm-l] Problema com inteiros gaussianos

2002-09-09 Por tôpico Augusto César Morgado

Nao podemos chegar ao infinito na reta real se os nossos passos  tiverem 
comprimento limitado (isto eh, existe um numero M tal que nossos 
passosnao podem ser maiores que M) e andarmos apenas sobre os numeros 
primos.
(por exemplo, suponha que voce nao possa dar passos maiores que 5. 
Andando nosprimos, voce pode começar no 2, passar para o 7, depois para 
o 11, depois para o 13, o 17, o 19, o 23.
Agora voce empaca. Do 23 para o primo seguinte o salto eh maior que 5.
A essencia do que o autor quer dizer eh que, por maior que seja a 
limitaçao dos seus passos (5, no meu exemplo), chega uma hora que voce 
empaca. )
Isso eh simplesmente uma outra maneira de dizer  que ha vazios 
arbitrariamente grandes entre os primos. (ou seja, tomando o 5, 
encontramos um vazio de tamanho maior que 5 nos primos: entre o 23 e o 
29.  Tomando o 7, encontrariamos um vazio de tamanho maior que 7; nao eh 
dificil perceber que esse vazio se dah entre o 31 e o 39. Tomando o 20, 
encontrariamos um vazio maior que 20 etc).
A prova eh simples. Um vazio de tamanho k ocorre entre (k+1)!+2 e 
(k+1)!+k+1. Com efeito, nenhum desse numeros eh primo, pois  (k+1)!+2 eh 
divisivel por 2(e eh maior que 2)  , (k+1)!+3 eh divisivel por 3 (e eh 
maior que 3) etc

Jackson Graziano wrote:

Caros amigos,

Como devo interpretar o enunciado abaixo? Nao consegui entender como são as
regras dele pra andar na reta real...

One cannot walk to infinity on the real line if one uses steps of bounded
length and steps on the prime numbers. This is simply a restatement of the
classic result that there are arbitrarily large gaps in the primes. The
proof is simple: a gap of size k is given by (k + 1)! + 2, (k + 1)! + 3,...,
(k + 1)! + (k,+1).
But the same problem in the complex realm is unsolved. More precisely, an
analogous question asks whether one can walk to infinity in Z[i], the
Gaussian integers, using the Gaussian primes as stepping stones, and taking
steps of bounded length

Obrigado,

Jackson Graziano

=
Instruções para entrar na lista, sair da lista e usar a lista em
http://www.mat.puc-rio.br/~nicolau/olimp/obm-l.html
O administrador desta lista é [EMAIL PROTECTED]
=




=
Instruções para entrar na lista, sair da lista e usar a lista em
http://www.mat.puc-rio.br/~nicolau/olimp/obm-l.html
O administrador desta lista é [EMAIL PROTECTED]
=



Re: [obm-l] funcao

2002-09-07 Por tôpico Augusto César Morgado

Uma funçao eh uma correspondencia. A funçao que associa a cada real o 
seu dobro pode ser descrita como associando a cada real x o valor de 2x 
(ou seja, f(x)=2x), mas pode ser descrita como associando a cada real y 
o valor de 2y (ou seja, f(y) = 2y), etc.
A funçao, ou seja a correspondencia, eh f.  
f(x) eh o valor que f associa a x.
Deu pra entender?
Se deu, otimo. Agora, ca entre nos e que os muito rigorosos nao nos 
leiam. Todo mundo fala

seja uma funcao f(x) querendo dizer
seja uma funçao f que a cada x associa f(x)

Portanto, nao se preocupe em demasia com esse erro, que eh na verdade apenas um abuso 
de linguagem.
Morgado



adr.scr.m wrote:

li num livro e gostaria de saber porque eh 
errado falar  seja uma funcao f(x),e o 
certo eh  seja uma funcao f  .
[]'s.
Adriano.

 
__
AcessoBOL, só R$ 9,90! O menor preço do mercado!
Assine já! http://www.bol.com.br/acessobol


=
Instruções para entrar na lista, sair da lista e usar a lista em
http://www.mat.puc-rio.br/~nicolau/olimp/obm-l.html
O administrador desta lista é [EMAIL PROTECTED]
=




=
Instruções para entrar na lista, sair da lista e usar a lista em
http://www.mat.puc-rio.br/~nicolau/olimp/obm-l.html
O administrador desta lista é [EMAIL PROTECTED]
=



Re: [obm-l] listas de treinamento interncionais

2002-09-07 Por tôpico Augusto César Morgado

Acesse  www.obm.org.br

basketboy_igor wrote:

Gostaria de ser informando de sites ou receber lista de 
treinamentos intenacionais p/ IMO, USAMO, Ibero, torneio 
das cidades e outras olimpíadas internacionais de 
matemática, ou de sites que contenham questões vaiadas 
de matemática vindas de várias partes do mundo.


 
__
AcessoBOL, só R$ 9,90! O menor preço do mercado!
Assine já! http://www.bol.com.br/acessobol


=
Instruções para entrar na lista, sair da lista e usar a lista em
http://www.mat.puc-rio.br/~nicolau/olimp/obm-l.html
O administrador desta lista é [EMAIL PROTECTED]
=




=
Instruções para entrar na lista, sair da lista e usar a lista em
http://www.mat.puc-rio.br/~nicolau/olimp/obm-l.html
O administrador desta lista é [EMAIL PROTECTED]
=



Re: [obm-l] ???

2002-09-07 Por tôpico Augusto César Morgado



Ha alguma coisa errada no problema 2.
Em um polinomio de coeficientes inteiros, P(a) - P(b) eh divisivel por a-b
(a, b inteiros, naturalmente).
Entao, 247 - 17 = 230 deveria ser divisivel por 32 - 21 = 11.

Eder wrote:

  
  
  Gostaria de ajuda nestes problemas:
  
  1)Encontre todas as solues reais de
 cosx+(cosx)^5+cos7x=3.
  
  Bom,eu tranformei cosx+cos7x em produto,depois
sa  fazendo simplificaes para ficar somente com cosx.Substituindo cosx
po  m,cheguei a um polinmio bem "estranho".Foi fcil checar que m=1  ==cosx=1==x=2kpi
 soluo (na verdade  fcil de ver isso no  enunciado),porm no soube
mais o que fazer depois disso,para ver se h outras  solues.
  
  2)Considere um polinmio de coeficientes
 inteiros.Sabe-se que p(21)=17,p(32)=-247 e p(37)=33.Prove que se para  algum
N tivermos p(N)=51.Ento N=26.
  
  
  Obrigado pela ajuda.
  
  Eder
  
  
  
  
  
  


Re: [obm-l] ???

2002-09-07 Por tôpico Augusto César Morgado



OK, o sinal de - estava ilegivel. Obrigado ao Edilon que chamou a atenao.
Morgado

Eder wrote:
003701c256cf$da426ac0$3758fea9@Eder">
  
  Vou checar se no distorci o problema.Ele
caiu na  olimpada interna do meu colgio.
  

- Original Message - 

From:
AugustoCsar Morgado


To:
[EMAIL PROTECTED]


Sent: Saturday, September 07, 2002 9:07PM

Subject: Re: [obm-l] ???


Ha alguma coisa errada no problema 2.
Em um polinomio decoeficientes inteiros, P(a) - P(b) eh divisivel por
a-b (a, b inteiros,naturalmente).
Entao, 247 - 17 = 230 deveria ser divisivel por32 - 21 = 11.

Eder wrote:

  
  
  Gostaria de ajuda nestes  problemas:
  
  1)Encontre todas as solues reais
de  cosx+(cosx)^5+cos7x=3.
  
  Bom,eu tranformei cosx+cos7x em produto,depois
 sa fazendo simplificaes para ficar somente com cosx.Substituindo
cosx po  m,cheguei a um polinmio bem "estranho".Foi fcil checar que
m=1  ==cosx=1==x=2kpi  soluo (na verdade  fcil de ver isso
no  enunciado),porm no soube mais o que fazer depois disso,para ver
se h  outras solues.
  
  2)Considere um polinmio de coeficientes
 inteiros.Sabe-se que p(21)=17,p(32)=-247 e p(37)=33.Prove que se para
 algum N tivermos p(N)=51.Ento N=26.
  
  
  Obrigado pela ajuda.
  
  Eder
  
  
  
  
  
  
  
  
  


Re: [obm-l] 0 é imaginário puro?

2002-09-06 Por tôpico Augusto César Morgado

Pegue qualquer livro decente (por exemplo, Churchill Complex Variables 
 ou Boas Mathematical Methods in the Physical Sciences) e você verá 
que eles definem imaginário puro como um complexo x+yi cuja parte real x 
é igual a 0.
Agora, ser ou não ser não é uma questão de pensamentos. A questão é usar 
os nomes no sentido que a comunidade matemática emprega (afinal, é a 
comunidade que define a norma culta).
Em relação ao primeiro livro que citei, estimo que 3 em cada 4 
matemáticos com mais de 40 anos de idade tenham-no lido.

marcelo oliveira wrote:

 Esta dúvida surgiu durante a última prova de matemática da AFA.

 Finalmente, pode-se considerar 0 como imaginário puro?

 Claramente a primeira idéia é não considerar 0 como imaginário puro, 
 por pensamentos puramente algébricos.

 Entretanto pense no plano imaginário (plano de Argand-Gauss) e note 
 que 0 (a origem do sistema) pertence ao eixo imaginário (e real 
 também?!).

 Gostaria também de saber uma justificativa (se houver, caso não seja 
 uma simples convenção) para o fato de 0 ser ou não ser imaginário puro.


 Até mais,
 Marcelo Rufino de Oliveira

 _
 MSN Photos is the easiest way to share and print your photos: 
 http://photos.msn.com/support/worldwide.aspx

 =
 Instruções para entrar na lista, sair da lista e usar a lista em
 http://www.mat.puc-rio.br/~nicolau/olimp/obm-l.html
 O administrador desta lista é [EMAIL PROTECTED]
 =




=
Instruções para entrar na lista, sair da lista e usar a lista em
http://www.mat.puc-rio.br/~nicolau/olimp/obm-l.html
O administrador desta lista é [EMAIL PROTECTED]
=



Re: [obm-l] Ajuda em fatorial.

2002-09-06 Por tôpico Augusto César Morgado



1) m(m-1)!=m!
m!/(m+1)!=1/(m+1)
A equaao fica (m+3)/[(m-2)(m+1)] = 6/35
6m^2 -41m -117=0
A unica soluao inteira eh 9.

2)a) [(m + 2)! - (m + 1)!] m! = 24 
(m+1)![(m+2)-1]m!=24^2
(m+1)! (m+1) m! = 24^2
(m+1)! (m+1)! = 24^2
(m+1)! = 24
m+1 = 4
m=3
b) (m + 3)! + (m + 2)! = 6
(m + 3)! - (m + 2)!
Divida numerador e denominador por (m+2)!
Fica
[(m+3)+1]/ [(m+3)-1] = 6
(m+4)/(m+2) = 6
Esta equao no possui soluo inteira.

c)?

4) a) (n+2)!
b) (n-7)!

3) n inteiro e tambm
a) maior que ou igual a 0
b)  maior que ou igual a 5
c)  maior que ou igual a -1

Sharon Guedes wrote:
[EMAIL PROTECTED]">
  Ol pessoal ! 
  Algum poderia me ajudar nestas questes?
  1) (UnB)Sendo m . (m + 3) . (m - 1)! = 6 
e m  0, o valor de m :
  (m - 2) . (m + 1)!  35
  a) 9
  b) 12
  c) 15
  d) 18
  2)(URCAMP)Resolva as equaes:
  a) [(m + 2)! - (m + 1)!] m! = 24 
  b) (m + 3)! + (m + 2)! = 6
  (m + 3)! - (m + 2)!
  c) (n + 2)! + (n + 1)! = 21
  1 . 2 . 3 ... (n - 3) (n - 2) (n - 1)!
  3)(URCAMP)D o domnio de cada uma destas expreses:
  a) n!
  b) (n - 5)!
  c) (n + 1)! (n + 1)!
  (URCAMP)Escreva os produtos empregando a notao fatorial.
  a)(n + 2) . (n + 1) . n . (n - 1) . ... . 2 .1
  b)(n -7). (n - 8). (n - 9). ... . 2 .1
  At. Sharon.
  
  
  
  Yahoo!
PageBuilder
 - O super editor para criao de sites:  grtis, fcil e rpido. 
  
  
  


[Fwd: Re: [obm-l] Números Complexos]

2002-09-02 Por tôpico Augusto César Morgado



5) Representando, no plano, as raizes complexas da equacao z^3 + 8 = 0,
obtem-se um triangulo. Calcule a area desse triangulo.
z^3 = -8
modulo de z = 2
As imagens das raizes da equaao sao vertices de um triangulo equilatero
inscrito num circulo de centro na origem e raio 2. O lado vale 2raiz de3
e a area vale 3raiz de 3.

6) (x+yi)^2 = x-yi
x^2-y^2 +2xyi = x-yi
x^2-y^2 = x e 2xy = -y
A segunda equaao dah y=0 ou x = -(1/2)
Substituindo na primeira, x=0 ou x=1 no primeiro caso, y = (+-) [raiz de3]/2
no segundo.
Ha quatro soluoes: 0 ;  1 ;  - 1/2 + (sqrt3)/2  ;  - 1/2 - (sqrt3)/2

Desde quando 0 nao eh complexo?
Morgado

 Original Message 

  

  From: 
  - Mon Sep 02 20:06:02 2002


  X-UIDL: 
  F5;!!GlU!!\?e"!I:m!!


  X-Mozilla-Status: 
  0001


  X-Mozilla-Status2: 
  


  Return-Path: 
  [EMAIL PROTECTED]


  Received: 
  from sucuri.mat.puc-rio.br (sucuri.mat.puc-rio.br [139.82.27.7])	by
trex.centroin.com.br (8.12.5/8.12.1) with ESMTP id g82LZC4B009660	for [EMAIL PROTECTED];
Mon, 2 Sep 2002 18:35:12 -0300 (BRT)


  Received: 
  (from majordom@localhost)	by sucuri.mat.puc-rio.br (8.9.3/8.9.3)
id SAA20705	for obm-l-MTTP; Mon, 2 Sep 2002 18:31:44 -0300


  Received: 
  from smtp.ieg.com.br (stone.protocoloweb.com.br [200.226.139.11])	by
sucuri.mat.puc-rio.br (8.9.3/8.9.3) with ESMTP id SAA20701	for [EMAIL PROTECTED];
Mon, 2 Sep 2002 18:31:42 -0300


  Received: 
  from localhost ([EMAIL PROTECTED] [200.158.118.125])	by
smtp.ieg.com.br (IeG relay/8.9.3) with SMTP id g82LSDfE067536	for [EMAIL PROTECTED];
Mon, 2 Sep 2002 18:28:13 -0300 (BRT)


  From: 
  Tonik [EMAIL PROTECTED]


  To: 
  [EMAIL PROTECTED]


  Date: 
  Mon, 02 Sep 2002 18:31:23 -0300


  X-Priority: 
  3 (Normal)


  Organization: 
  Tonik


  In-Reply-To: 
  003201c2529c$f2084800$0200a8c0@dois


  Message-Id: 
  BA4WNB9ED86BAVRE0VQLYSC7HC7RM.3d73d8ab@localhost


  Subject: 
  Re: [obm-l] Nmeros Complexos


  MIME-Version: 
  1.0


  Content-Type: 
  text/plain; charset="iso-8859-1"


  X-Mailer: 
  Opera 6.04 build 1135


  Sender: 
  [EMAIL PROTECTED]


  Precedence: 
  bulk


  Reply-To: 
  [EMAIL PROTECTED]


  X-UIDL: 
  F5;!!GlU!!\?e"!I:m!!


  Status: 
  U

  



02/09/02 13:22:18, Gabriel Prgola  wrote:

E a pessoal,

Gostaria de ver a resoluo destes problemas de nmeros complexos que no
consegui fazer:

Sao exercicios simples, q vou fazer mais para me exercitar, pois sao mais 
trabalhosos do q desafiantes... cheque as contas!

1) Obtenha o argumento de sen 40 + i cos 40

obviamente, 40

2) Determine o menor valor inteiro e positivo de n para o qual (1 + i
sqrt[3])^n  um numero real

para (1 + i*sqrt(3))^n ser real, seu argumento devera ser 0 ou 180
ou k180, k E Z, passando (1+isqrt(3)) para a forma trigonometrica, 
temos:
modulo = sqrt(1^2 + sqrt(3)^2) = sqrt(1+3) = 2
argumento = arccos(1/2) = 60

entao temos (2*(cos60+isen60))^n =
= 2^n*(cos(60*n)+isen(60*n)
para que o argumento (60*n) de 0 ou 180 com n0, n E Z:

60*n=360, n=6
6
0*n=180, n=3

Logo a resposta eh 3.

3) Determine o menor valor inteiro e positivo de n para o qual (1 + i
sqrt[3])^n  um numero real positivo.

a mesma coisa, s que agora 180 nao serve (pois eh real negativo)
60*n=360, n=6

4) Obtenha as raizes complexas das equacoes:
a) x^5 = 1
b) x^6 = 1

x^5 = 1
x= raizquintupla(1*(cos0+isen0))
x= cos(0/5 + 360k/5) + isen(0/5 + 360k/5), 0=k5, k E Z

as raizes:
x= cos0+isen0 = 1 (nao eh complexa)
x= cos72+isen72
x= cos144+isen144
x= cos216+isen216
x= cos288+isen288

5) Representando, no plano, as raizes complexas da equacao z^3 + 8 = 0,
obtem-se um triangulo. Calcule a area desse triangulo.

z^3=-8 tem 3 raizes, por 3 ser impar, uma das raizes eh real
z= raizcubica(-8)
z= raizcubica( 8*(cos180+isen180) )
z= 2*(cos(180/3+360k/3)+isen(180/3+360k/3)), 0=k3
z= 2*(cos(60+120k)+isen(60+120k))
as 
raizes:
k=0, z=2*(cos60+isen60) = 2*(1/2 + i*sqrt(3)/2) = 1+i*sqrt(3)
k=1, z=2*(cos180+isen180) = 2*(-1 + i*0) = -2
k=2, z=2*(cos300+isen300) =
(sabe-se que 1+isqrt(3) eh raiz, entao seu conjugado, 1-isqrt(3) eh a 
terceira raiz)

entao temos os pontos do triangulo ABC no plano complexo:
A(1,  sqrt(3))
B(-2, 0)
C(1,  -sqrt(3))

Seja D a matriz:
|Ax Ay 1| 
|Bx By 1|
|Cx Cy 1|

Area = modulo do determinante de D sobre 2
Area = |sqrt(3)+2sqrt(3)-(-sqrt(3)-2sqrt(3))|/2
Area = 3sqrt(3)

6) A quantidade de numeros complexos que tem o seu quadrado igual ao seu
conjugado ?

Seja z um numero complexo, vc quer a qtde de n complexos que
z^2 = conjugado de z

pela forma trigonometrica, seja m o modulo e a o argumento:

m^2*(cos(2a)+isen(2a)) = m*(cos(a)-isen(a))
sabemos 

Re: [obm-l] Existe??

2002-09-01 Por tôpico Augusto César Morgado



F(x) = x-cosx eh continua em [0, pi/2].
F(0) = -1 e F(pi/2) = pi/2.
Logo, pelo teorema de Bolzano...


e isso mesmo wrote:
[EMAIL PROTECTED]">
  
  Companheiros, ajudem-me:
  
  -Mostre que existe um nmero x entre 0 e Pi/2 tal que x=cos x.
  
  Obrigado
  
  Aproveite melhor a Web. Faa o download GRTIS do MSN Explorer : 
http://explorer.msn.com.br/intl.asp#po
  
  
  
  
  


Re: [obm-l] Problema de Trigonometria

2002-08-31 Por tôpico Augusto César Morgado



cosx. cosx - cotx = cosx. cosx - (cosx/senx) = (cosx cosx senx - cosx)/senx
= cosx (cosx senx - 1)/senx
senx senx - tan x = senx senx - (senx/cosx) =( senx senx cosx - senx )/cosx
= senx (senx cosx -1) /cosx
Divida e pronto.
Edmilson wrote:
000f01c250dd$c06bf410$a81fffc8@edmilson">
  
  
  Caros amigos,
  
  De uma lista de 30 problemas de Trigonometria s no consegui resolver
 este.
  
  Simplificando a expresso 
  
  
  Fiz no Maple e sei que a resposta 
  
  
  Me ajudem por favor.
  
  Atenciosamente,
Edmilson
  [EMAIL PROTECTED]
  
  
  
  
  


Re: [obm-l] esclarecimento

2002-08-31 Por tôpico Augusto César Morgado



Eh 3/8. Adoraria saber qual o raciocinio que conduziu a resposta 1/4.
Supondo naturalmente as moedas nao-tendenciosas.
Morgado

[EMAIL PROTECTED] wrote:
[EMAIL PROTECTED]">
Ao jogar trs moedas, qual a probabilidade de dar duas caras e uma coroa?
 Alguns colegas acham que  1 / 4 outros acham que  3 / 8. Por que a confuso?
  possvel as duas respostas estarem corretas?
  
  
  
  


Re: [obm-l] Neperiano

2002-08-30 Por tôpico Augusto César Morgado

Leia o livro do Elon Logaritmos editado pela SBM na Coleção do 
Professor de Matemática

[EMAIL PROTECTED] wrote:

Será que alguém da lista , poderia me dizer mais sobre os logaritmos neperianos,
ou me indicar um site , onde eu possa saber algo sobre o assunto ?
Abraço.
Rick.


  
 |-=Rick-C.R.B.=- |
 |ICQ 124805654   |
 |e-mail [EMAIL PROTECTED]  |
  


--
Use o melhor sistema de busca da Internet
Radar UOL - http://www.radaruol.com.br



=
Instruções para entrar na lista, sair da lista e usar a lista em
http://www.mat.puc-rio.br/~nicolau/olimp/obm-l.html
O administrador desta lista é [EMAIL PROTECTED]
=




=
Instruções para entrar na lista, sair da lista e usar a lista em
http://www.mat.puc-rio.br/~nicolau/olimp/obm-l.html
O administrador desta lista é [EMAIL PROTECTED]
=



Re: [obm-l] interpretação..

2002-08-28 Por tôpico Augusto César Morgado



2) Sejam 2p+1, 2p+3,..., 2p+2n-1 os n nmeros. 
A soma eh (2p+1+2p+2n-1)n/2 = (2p+n)n 
Portanto, (2p+n)n eh um produto de inteiros que eh igual a 7^3 e, embora
eu nao saiba o que acontece com 2p+n, n eh positivo.
Os unicos produtos de dois inteiros, um dos quais eh positivo, que dao 7
^3=343 sao
1* 343, 7*49, 49*7, 343*1.
Faa os quatro casos e termine.

Augusto Csar Morgado wrote:
[EMAIL PROTECTED]">   3)
Sejam r o racional nao-nulo e x o irracional.
 Se xr fosse um racional p, teriamos xr=p e x = p/r seria racional, o que 
eh absurdo.
 1a) supondo as equaoes do primeiro grau, cada equaao representa um plano. 
Sao tres planos com exatamente um ponto comum.
 1b) Dois planos, nao-paralelos. Portanto o sistema representa uma reta.
Leia A Matematica do Ensino Medio, de Elon L. Lima.
  
  [EMAIL PROTECTED]
 wrote:
  [EMAIL PROTECTED]">
 1)Como se interpreta geomtricamente um sistema na vriveis x, y e z, que 
seja possvel e determinado supondo que esse sistema tenha tres equaes?
  Qual a interpretao geomtrica para o seguinte sistema??
  x+y +2z=10
  x+y +z=4
  2)uma soma finita de numeros inteiros consecutivos, impares, positivos
ou negativos,  igual a 7^3. Determine os termos dessa soma.
  3)Prove que o produto de um numero racional no nulo por um irracional
 um numero irracional.
   Muito obrigado a quem esclarecer.
   Korshini







Re: [obm-l] Filhos

2002-08-27 Por tôpico Augusto César Morgado



h-1=m
h=2(m-1)
Resolvendo, m=3 e h=4
m+h=7

Hely Jr. wrote:
000801c24db8$0516d9a0$[EMAIL PROTECTED]">
  
  
  Vejam esta questo. 
  
  "Um casal tem filhas e filhos. Cada filho
tem um  numero de irmos igual ao numero de irms.
  Cada filha tem um numero de irmos igual
ao dobro  do numero de irms. Qual o total de filhos e filhas do  casal?"
  
  
  
  


Re: [obm-l] Re:

2002-08-27 Por tôpico Augusto César Morgado



No adianta nada acrescentar a condio irredutvel.. Se p/q irredutvel
satisfaz e q diferente de 1 , escolha um n muito, muito grande, de forma
que (np-1)/nq continue satisfazendo a desigualdade e tal que n seja multiplo
de q. (np-1)/nq sera irredutivel.

Johann Peter Gustav Lejeune Dirichlet wrote:
[EMAIL PROTECTED]">
   Eu acho que a fraao e irredutivel,nao? 
   Eder [EMAIL PROTECTED] escreveu:  
  

Esse problema apareceu na primeira ou
na segunda Eureka,se no me engano e o enunciado  assim mesmo.

  
- Original Message - 
  
From:
Augusto Csar Morgado
  
  
To:
[EMAIL PROTECTED]
  
  
Sent: Monday, August 26, 2002 7:45 PM
  
Subject: [obm-l] Re: 
  
  
Eh isso mesmo?
A resposta eh nao existe. Claro, se 7/10 
p/q  11/15 entao  7/10  np/nq
 11/15 e se um q satisfaz, todos os multiplos satisfarao.
  
Eder wrote:
  002301c24d48$fd9241c0$f1f3fea9@Eder" type="cite">


Ser que algum poderia me ajudar
neste problema:

Se p e q so inteiros positivos
tais que 7/10  p/q  11/15 ,qual o maior valor que q pode assumir?


Obrigado.





Yahoo! PageBuilder
 - O super editor para criao de sites:  grtis, fcil e rpido. 





Re: [obm-l] interpretação..

2002-08-27 Por tôpico Augusto César Morgado



3) Sejam r o racional nao-nulo e x o irracional.
Se xr fosse um racional p, teriamos xr=p e x = p/r seria racional, o que
eh absurdo.
1a) supondo as equaoes do primeiro grau, cada equaao representa um plano.
Sao tres planos com exatamente um ponto comum.
1b) Dois planos, nao-paralelos. Portanto o sistema representa uma reta. Leia
A Matematica do Ensino Medio, de Elon L. Lima.

[EMAIL PROTECTED] wrote:
[EMAIL PROTECTED]">
1)Como se interpreta geomtricamente um sistema na vriveis x, y e z, que
seja possvel e determinado supondo que esse sistema tenha tres equaes?
 Qual a interpretao geomtrica para o seguinte sistema??
 x+y +2z=10
 x+y +z=4
 2)uma soma finita de numeros inteiros consecutivos, impares, positivos ou
negativos,  igual a 7^3. Determine os termos dessa soma.
 3)Prove que o produto de um numero racional no nulo por um irracional 
um numero irracional.
  Muito obrigado a quem esclarecer.
  Korshini
  
  
  


Re: [obm-l] (sem assunto)

2002-08-25 Por tôpico Augusto César Morgado

3) n = 4k
 A partir daqui, = significa congruo modulo 10
1^n = 1 (mod 10)
  2^n  = 16 ^k = 6^k = 6
3^n = 81^k = 1^k = 1
4^n = 254^k = 6^k = 6
5^n = 5
6^n = 6
7^n = 2401^k = 1
8^n = 4096^k = 6^k = 6
9^n = 81^(2k) = 1^(2k) = 1
A soma eh congrua a 1+6+1+6+5+6+1+6+1 = 33 que eh congruo a 3.
Resposta: 3
Bruno F. C. Leite wrote:

 At 23:02 24/08/02 -0400, you wrote:

 Olá rapaziada...vai ai um..se alguem puder ajudar.
 1)Prove que existem infinitos primos p tais que sejam congruos a 3 
 modulo 4.


 Acho que já madei uma solução deste problema para a lista, dê uma 
 olhada nos arquivos!

 2)Qual o resto da divisão euclidiana de s=1^5+2^5+3^5+...+99^5+100^5 
 por 4?? Justifique.


 Observe que você pode ignorar os números pares da soma: todos eles 
 (2^5, 4^5, etc) são multiplos de 4. Para os impares, observe que 
 (4k+1)^5+(4k+3)^5 é sempre multiplo de 4...

 Bruno Leite
 http://www.ime.usp.br/~brleite


 3)Se n é um multiplo de 4, qual o resto da divisão de 
 1^n+2^n++8^n+9^n por 10?
Valeu
 = 

 Instruções para entrar na lista, sair da lista e usar a lista em
 http://www.mat.puc-rio.br/~nicolau/olimp/obm-l.html
 O administrador desta lista é [EMAIL PROTECTED]
 = 



 =
 Instruções para entrar na lista, sair da lista e usar a lista em
 http://www.mat.puc-rio.br/~nicolau/olimp/obm-l.html
 O administrador desta lista é [EMAIL PROTECTED]
 =




=
Instruções para entrar na lista, sair da lista e usar a lista em
http://www.mat.puc-rio.br/~nicolau/olimp/obm-l.html
O administrador desta lista é [EMAIL PROTECTED]
=



Re: [obm-l] Duas questões do IME.

2002-08-25 Por tôpico Augusto César Morgado



2) Se sao n clubes nao fluminenses, o total de pontos eh 8+kn. Mas isso eh
igual ao total de jogos, Cn+2,2 = (n+2)(n+1)/2.
Igualando, k = (n+3)/2 - 7/n.
2k eh inteiro.
2k= n+3 - 14/n.
Entao n so pode ser 1 ou 2 ou 7 ou 14.
n=1 e n=2 sao absurdos pois k seria negativo.
Logo, n=7 ou n=14.

Bem, agora vou propor outro problema. Esta soluao estah correta ou nao?


[EMAIL PROTECTED] wrote:

 Ol pessoal da lista,gostaria de uma ajuda nessas duas questes
  
 do IME. 
  1) 12 cavaleiros esto sentados em torno de uma mesa redonda
.Cada um dos doze cavaleiros considera seus dois vizinhos como rivais.Deseja-se
formar um grupo de 5 cavaleiros para libertar uma princesa.Nesse grupo no
poder haver cavaleiros rivais .Determine de quantas maneiras  possvel
escolher esse grupo.
  
  2) Dois clubes do Rio de Janeiro participaram de um campeonato
nacional de futebol de salo onde cada vitria valia 1 ponto,cada empate
meio ponto e cada derrota zero ponto.Sabendo que cada participante enfrentou
todos os outros apenas uma vez,que os clubes do Rio de Janeiro totalizaram,em
conjunto, 8 pontos e que cada um dos outros clubes alcanou a mesma quantidade
  k de pontos, determine a quantidade de clubes que participou do
torneio.
  
  Um abrao,
  Bruno Moss.
  
  
  
  


Re: [obm-l] Infinitos

2002-08-25 Por tôpico Augusto César Morgado



Eh de Dirichlet.

Carlos Victor wrote:
[EMAIL PROTECTED]">
 Ol Rubens ,
  
 Acredito que algum j demonstrou isto aqui .Geralmente estas provas
so por absurdo .Suponha que exista uma quantidade finita de primos
desta forma .Considere os primos da forma dada : p1, p2 , p3 , ...,pr e
considere o nmero 
 K = 4p1.p2.p3. pr - 1 = 4(p1.p2.p3...pr -1 ) + 3 . Observe que Kpi
e  composto e deve ter fatores primos da forma 4s+1 ou 4s+3 , e j
que multiplicando fatores da forma 4s+1 teremos fatores da forma 4s+1
, conclumos que K deve ter pelo menos um fator da forma 4s+3 . Isto
 um absurdo j que este fator dever dividir a unidade , ok ? 
  
 Na verdade existe um teorema geral que diz : Se a e b so inteiros  positivos
primos entre si , ento existe infinitos primos da forma an+b . No
me lembro de quem  este teorema .
  
 []s Carlos Victor
  
  
  
 At 15:36 25/8/2002 -0300, Rubens Vilhena wrote:
  
 Ol pessoal
 
 1) Demonstrar que existem infinitos primos da forma 4n+3, com n inteiro.
 
 Ok!

 Aproveite melhor a Web. Faa o download GRTIS do MSN Explorer : 
http://explorer.msn.com.br/intl.asp#po







Re: [obm-l] ???

2002-08-19 Por tôpico Augusto César Morgado



2) Se n eh par  (8^n )*19+17 eh congruo,
modulo 3, a (-1)^n + 2 = 1+2 = 3 eh congruo a 0, ou seja, eh multiplo de
3.
Se n eh da forma 4k+1, a congruencia modulo 13 dah
 (8^n )*19+17 congruo a (8^4k)*8*19+17
 congruo a (64^2k)*8*6+4 congruo a [(-1)^2k] * 48 + 4 congruo a 1*48+4=52
congruo a 0, ou seja, eh multiplo de 13.
Se n eh da forma 4k+3, a congruencia modulo 5 dah
 (8^n )*19+17 congruo a  (8^4k)*512*19+17
 congruo a (64^2k)*2*4+2 congruo a  [(-1)^2k] * 8 + 2 congruo a 1*8+2=10
congruo a 0, ou seja, eh multiplo de 5.


Eder wrote:
003f01c247c8$f540ef60$8905fea9@Eder">
  
  
  Ol,
  
  A vo alguns problemas que no estou
 conseguindo resolver:
  
  i)Encontre todas as solues inteiras
de a-3ab-a+b  = 0.
  ii)Mostre que (8^n )*19+17  composto
para qualquer  inteiro no-negativo n.
  
  Grato por quaisquer comentrios.
  
  Eder
  
  
  
  


Re: [obm-l] Integrais pesadas!

2002-08-19 Por tôpico Augusto César Morgado



Pea aa SBM.

Fernando Henrique Ferraz P. da Rosa wrote:
[EMAIL PROTECTED]">
 Voc por um acaso no sabe se h alguma verso online desse artigo
 para consulta? Procurei a 'Matematica Universitaria' nas bibliotecas da
USP  aqui em SP mas nenhuma tinha os ultimos numeros dessa revista. 
  
At 09:25 8/18/2002 -0300, you wrote: 
  
  Ha um artigo do professor Daniel Cordeiro no ultimo
numero da Matematica  Universitaria. 

iver wrote: 
Ol, ser qua algum da lista poderia mostrar
um mtodo para se calcular  inegrais como sin(x/(x+1)) e semelhantes ?? (x^x
,...) 
  
agradeo antecipadamente por qualquer resposta. 
por favor, se puderem me indiquem livros onde eu possa estudar sobre isso... 
  
  
  
  
"... a perfect formulation of a problem is already half 
 its solution." 
 David Hilbert. 
- 
[]'s 
Fernando Henrique Ferraz Pereira da Rosa 
USP, IME, Estatstica 
http://www.linux.ime.usp.br/~feferraz
  
  ---Outgoing mail is certified Virus Free.Checked by AVG anti-virus system (http://www.grisoft.com).Version: 6.0.381 / Virus Database: 214 - Release Date: 8/2/2002
  
  
  
  


Re: [obm-l] ???

2002-08-19 Por tôpico Augusto César Morgado



Eh claro que o numero eh maior que 17. Portanto, nao pode ser igual nem a
3, nem a 5 nem a 13. Logo, sendo multiplo de um desses serah composto.

Augusto Csar Morgado wrote:
[EMAIL PROTECTED]">   2)
Se n eh par  (8^n )*19+17 eh congruo, modulo
3, a (-1)^n + 2 = 1+2 = 3 eh congruo a 0, ou seja, eh multiplo de 3.
 Se n eh da forma 4k+1, a congruencia modulo 13 dah
  (8^n )*19+17 congruo a (8^4k)*8*19+17
  congruo a (64^2k)*8*6+4 congruo a [(-1)^2k] * 48 + 4 congruo a 1*48+4=52 
congruo a 0, ou seja, eh multiplo de 13.
 Se n eh da forma 4k+3, a congruencia modulo 5 dah
  (8^n )*19+17 congruo a  (8^4k)*512*19+17
  congruo a (64^2k)*2*4+2 congruo a  [(-1)^2k] * 8 + 2 congruo a 1*8+2=10 
congruo a 0, ou seja, eh multiplo de 5.
  
  
 Eder wrote:
  003f01c247c8$f540ef60$8905fea9@Eder">


Ol,

A vo alguns problemas que no estou 
 conseguindo resolver:

i)Encontre todas as solues inteiras 
de a-3ab-a+b  = 0.
ii)Mostre que (8^n )*19+17  composto 
para qualquer  inteiro no-negativo n.

Grato por quaisquer comentrios.

Eder








Re: [obm-l] Integrais pesadas!

2002-08-18 Por tôpico Augusto César Morgado



Ha um artigo do professor Daniel Cordeiro no ultimo numero da Matematica
Universitaria.

iver wrote:
002601c2466c$674e6b40$f55da3c8@HUGO">
  
  
  Ol, ser qua algum da lista poderia
mostrar um  mtodo para se calcular inegrais como sin(x/(x+1)) e semelhantes??
(x^x  ,...)
  
  agradeo antecipadamente por qualquer
 resposta.
  por favor, se puderem me indiquem livros
onde eu  possa estudar sobre isso...
  
  
  
  


[Fwd: Re: Re: [obm-l] Problema das pesagens]

2002-08-18 Por tôpico Augusto César Morgado





 Original Message 

  

  From: 
  - Sat Aug 17 21:32:47 2002


  X-Mozilla-Status: 
  0001


  X-Mozilla-Status2: 
  


  Message-ID: 
  [EMAIL PROTECTED]


  Date: 
  Sat, 17 Aug 2002 21:32:44 -0300


  From: 
  Augusto Csar Morgado [EMAIL PROTECTED]


  User-Agent: 
  Mozilla/5.0 (Windows; U; Win98; en-US; rv:0.9.4.1) Gecko/20020508
Netscape6/6.2.3


  X-Accept-Language: 
  en-us


  MIME-Version: 
  1.0


  To: 
  [EMAIL PROTECTED]


  Subject: 
  Re: Re: [obm-l] Problema das pesagens


  References: 
  [EMAIL PROTECTED]


  Content-Type: 
  multipart/alternative; boundary="020809000308030109050701"

  



   Separe as bolas em tres grupos: 1234   5678   9/10/11/12
 Pese 1234 contra 5678
 a) Equilibrou
 A diferente estah no grupo 9/10/11/12
 Pese 567 contra 9/10/11
 Se equilibrar a diferente eh a 12 e voce gasta a terceira pesgem para descobrir 
se ela eh mais leve ou mais pesada que as normais.
 Se nao equilibrar, voce descobre que a diferente estah no grupo 9/10/11
e descobre, conforme o resultado da segunda pesagem, se ela eh mais leve
ou mais pesada que as normais; digamos que seja mais leve. A terceira pesagem 
serah
 9 contra 10: se equilibrar a diferente eh a 11; caso contrrio eh a mais 
leve entre 9 e 10.
 b) Nao equilibrou. 
 Digamos que 1234 seja mais pesado que 5678.
 Ou a diferente estah em 1234 e eh mais pesada que as normais, ou a diferente 
estah em 5678 e eh mais leve que as demais. Pese 125 contra 346.
 Se equilibrar, a diferente estah em 78 e eh mais leve. Pese 7 contra 8,
e a mais leve serah a diferente.
 Se nao equilibrar (digamos que 125 seja mais pesado que 346), voce descobre 
que a diferente ou estah no grupo 12 e eh mais pesada, ou estah no grupo 6
e eh mais leve. No primeiro caso, basta agora pesar 1 contra 2: a diferente 
eh a mais pesada das duas.


[EMAIL PROTECTED]
 wrote:

   Oi Morgado, De que modo voc consegue f(12) = 3 ??-- Mensagem original --
  
f(12) = 3Morgado[EMAIL PROTECTED] wrote:

  considere uma balana de dois pratos e n bolas sendo que uma delas possuipeso diferente (sem saber se a bola defeituosa  mais leve ou mais pesada)Determine a funo f:IN-IN tal que f(n)  o menor numero de pesagens
  
  
  suficientes
  

  para determinar a bola defeituosa, n=3.f(3) = f(4) = f(5) = 2f(6) = .. = f(11) = 3f(12) = .. = f(?) = 4algum consegue ver a lei de formao ?"Mathematicus nascitur, non fit"Matemticos no so feitos, eles nascem---Gabriel Haeserwww.gabas.cjb.net--Use o melhor sistema de busca da InternetRadar UOL - http://www.radaruol.com.br=Instrues para entrar na lista, sair da lista e usar a lista emhttp://www.mat.puc-rio.br/~nicol
au
/olimp/obm-l.htmlO administrador desta lista  [EMAIL PROTECTED]=
  
  =Instrues para entrar na lista, sair da lista e usar a lista emhttp://www.mat.puc-rio.br/~nicolau/olimp/obm-l.htmlO administrador desta lista  [EMAIL PROTECTED]=
  
  []'s, YuriICQ: 64992515--Use o melhor sistema de busca da InternetRadar UOL - http://www.radaruol.com.br
  
  
  
  
  


Re: [obm-l] Trigonomagia...(o retorno)

2002-08-17 Por tôpico Augusto César Morgado



cos2x + 3 sen2x + 8 senx cosx = cos 2x + 3 sen2x + 4 sen2x = cos2x + 7 sen2x
= cos 2x ( 1 + 7 tan2x) = cos 2x * (17/3) =
= mais ou menos ( 3/raiz de 13) * (17/3) = mais ou menos 17/raiz de 13 =
mais ou menos 17 * (raiz de 13)/13

Eduardo Casagrande Stabel wrote:
000f01c24599$96624290$0201a8c0@stabel">
  From: "leonardo mattos" [EMAIL PROTECTED]
  
Sera que alguem poderia conferir essa questao pra mim?!   tg2x=2/3 calcular y=cos2x + 3sen2x + 8senxcosxEu encontrei a seguinte resposta(17 raiz de 13 sobre 13),mas o gabarito daoutra.Sera que alguem poderia conferir pra mim?Um abrao,Leonardo

Chame z=2x.Pelas frmulas trigonomtricas 2*sen(x)*cos(x)=sen(2x), datg(z) = 2/3, donde sai quesen(z) = (2/13)*raiz(13)cos(z) = (3/13)*raiz(13).A expresso pedida valey=cos(z) + 3sen(z) + 4sen(z)=cos(z) + 7sen(z)=(17/13)*raiz(13).Eu marcaria a resposta (17/13)*raiz(13)...Eduardo.=Instrues para entrar na lista, sair da lista e usar a lista emhttp://www.mat.puc-rio.br/~nicolau/olimp/obm-l.htmlO administrador desta lista  [EMAIL PROTECTED]=






Re: [obm-l] Problema das pesagens

2002-08-17 Por tôpico Augusto César Morgado

f(12) = 3
Morgado

[EMAIL PROTECTED] wrote:

considere uma balança de dois pratos e n bolas sendo que uma delas possui
peso diferente (sem saber se a bola defeituosa é mais leve ou mais pesada)

Determine a função f:IN-IN tal que f(n) é o menor numero de pesagens suficientes
para determinar a bola defeituosa, n=3.

f(3) = f(4) = f(5) = 2
f(6) = .. = f(11) = 3
f(12) = .. = f(?) = 4

alguém consegue ver a lei de formação ?


Mathematicus nascitur, non fit
Matemáticos não são feitos, eles nascem
---
Gabriel Haeser
www.gabas.cjb.net


--
Use o melhor sistema de busca da Internet
Radar UOL - http://www.radaruol.com.br



=
Instruções para entrar na lista, sair da lista e usar a lista em
http://www.mat.puc-rio.br/~nicolau/olimp/obm-l.html
O administrador desta lista é [EMAIL PROTECTED]
=




=
Instruções para entrar na lista, sair da lista e usar a lista em
http://www.mat.puc-rio.br/~nicolau/olimp/obm-l.html
O administrador desta lista é [EMAIL PROTECTED]
=



Re: [obm-l] teorema de fermat generalizado ...

2002-08-16 Por tôpico Augusto César Morgado

Deve ser
a elevado a fi de m é congruo a 1, modulo m, se a e m sao relativamente 
primos
fi de m é a funçao tociente de Euler que da o numero de elementos de 1, 
2, ..., m que sao relativamente primos com n.

Jose Augusto wrote:

Qual teorema seria esse?
obrigaod.

=
Instruções para entrar na lista, sair da lista e usar a lista em
http://www.mat.puc-rio.br/~nicolau/olimp/obm-l.html
O administrador desta lista é [EMAIL PROTECTED]
=




=
Instruções para entrar na lista, sair da lista e usar a lista em
http://www.mat.puc-rio.br/~nicolau/olimp/obm-l.html
O administrador desta lista é [EMAIL PROTECTED]
=



Re: [obm-l] teorema de fermat generalizado errata..

2002-08-16 Por tôpico Augusto César Morgado

Leia-se m onde esta n.

Augusto César Morgado wrote:

 Deve ser
 a elevado a fi de m é congruo a 1, modulo m, se a e m sao 
 relativamente primos
 fi de m é a funçao tociente de Euler que da o numero de elementos de 
 1, 2, ..., m que sao relativamente primos com n.

 Jose Augusto wrote:

Qual teorema seria esse?
obrigaod.

 = 

 Instruções para entrar na lista, sair da lista e usar a lista em
 http://www.mat.puc-rio.br/~nicolau/olimp/obm-l.html
 O administrador desta lista é [EMAIL PROTECTED]
 = 





 =
 Instruções para entrar na lista, sair da lista e usar a lista em
 http://www.mat.puc-rio.br/~nicolau/olimp/obm-l.html
 O administrador desta lista é [EMAIL PROTECTED]
 =




=
Instruções para entrar na lista, sair da lista e usar a lista em
http://www.mat.puc-rio.br/~nicolau/olimp/obm-l.html
O administrador desta lista é [EMAIL PROTECTED]
=



Re: [obm-l] Duvida de Geometria(Triangulo)

2002-08-16 Por tôpico Augusto César Morgado

O triangulo dos pontos medios eh semelhante ao original na razao 1/2. 
Logo, sua area eh (1/2)^2= 1/4 da area do original.

leonardo mattos wrote:


   Digamos que eu tenha um triangulo ABC cujos vertices B,C estao 
 variando nos eixos X,Y respectivamente,sendo A fixo.É dito que 
 conforme sao variados B e C a area do triangulo ABC se mantem 
 constante.Eu poderia dizer que o triangulos formado pelos pontos 
 medios dos lados do triangulo ABC mantem sua area constante tambem?E 
 em caso afirmativo,por quê?
   ???Leonardo???


 _
 Converse com seus amigos online, faça o download grátis do MSN 
 Messenger: http://messenger.msn.com.br

 =
 Instruções para entrar na lista, sair da lista e usar a lista em
 http://www.mat.puc-rio.br/~nicolau/olimp/obm-l.html
 O administrador desta lista é [EMAIL PROTECTED]
 =




=
Instruções para entrar na lista, sair da lista e usar a lista em
http://www.mat.puc-rio.br/~nicolau/olimp/obm-l.html
O administrador desta lista é [EMAIL PROTECTED]
=



Re: [obm-l] Duvida de Geometria(Triangulo)

2002-08-16 Por tôpico Augusto César Morgado

Se a area de ABC eh constante, 1/4 dessa area eh constante. Portanto, a 
resposta eh sim.

leonardo mattos wrote:

 Na realidade oq eu quero saber é se eu poderia afirmar que variando as 
 coordenadas de dos vertices B e C mantendo fixo A a area do triangulo 
 formado pelos pontos medios dos lados de ABC manteria-se constante 
 sabendo q a area de ABC de mantem constante.


 From: Augusto César Morgado [EMAIL PROTECTED]
 Reply-To: [EMAIL PROTECTED]
 To: [EMAIL PROTECTED]
 Subject: Re: [obm-l] Duvida de Geometria(Triangulo)
 Date: Fri, 16 Aug 2002 10:32:47 -0300

 O triangulo dos pontos medios eh semelhante ao original na razao 1/2. 
 Logo, sua area eh (1/2)^2= 1/4 da area do original.

 leonardo mattos wrote:


   Digamos que eu tenha um triangulo ABC cujos vertices B,C estao 
 variando nos eixos X,Y respectivamente,sendo A fixo.É dito que 
 conforme sao variados B e C a area do triangulo ABC se mantem 
 constante.Eu poderia dizer que o triangulos formado pelos pontos 
 medios dos lados do triangulo ABC mantem sua area constante tambem?E 
 em caso afirmativo,por quê?
   ???Leonardo???


 _
 Converse com seus amigos online, faça o download grátis do MSN 
 Messenger: http://messenger.msn.com.br

 = 

 Instruções para entrar na lista, sair da lista e usar a lista em
 http://www.mat.puc-rio.br/~nicolau/olimp/obm-l.html
 O administrador desta lista é [EMAIL PROTECTED]
 = 





 = 

 Instruções para entrar na lista, sair da lista e usar a lista em
 http://www.mat.puc-rio.br/~nicolau/olimp/obm-l.html
 O administrador desta lista é [EMAIL PROTECTED]
 = 






 _
 Tenha você também um MSN Hotmail, o maior webmail do mundo: 
 http://www.hotmail.com/br

 =
 Instruções para entrar na lista, sair da lista e usar a lista em
 http://www.mat.puc-rio.br/~nicolau/olimp/obm-l.html
 O administrador desta lista é [EMAIL PROTECTED]
 =




=
Instruções para entrar na lista, sair da lista e usar a lista em
http://www.mat.puc-rio.br/~nicolau/olimp/obm-l.html
O administrador desta lista é [EMAIL PROTECTED]
=



Re: [obm-l] Numeros Complexos e Inversao

2002-08-15 Por tôpico Augusto César Morgado

Veja  A Matemática do Ensino Médio, volume 3, editado pela SBM.

leonardo mattos wrote:


 Sera que alguem poderia me ajudar a compreender melhor a inversao em 
 numeros complexos?!
 Nao estou conseguindo entender muito bem esta teoria, principalmente a 
 parte de preservação de angulos e tudo o mais...
  Um abraço,Leonardo



 _
 Tenha você também um MSN Hotmail, o maior webmail do mundo: 
 http://www.hotmail.com/br

 =
 Instruções para entrar na lista, sair da lista e usar a lista em
 http://www.mat.puc-rio.br/~nicolau/olimp/obm-l.html
 O administrador desta lista é [EMAIL PROTECTED]
 =




=
Instruções para entrar na lista, sair da lista e usar a lista em
http://www.mat.puc-rio.br/~nicolau/olimp/obm-l.html
O administrador desta lista é [EMAIL PROTECTED]
=



Re: [obm-l] 0,9999... = 1?

2002-08-15 Por tôpico Augusto César Morgado

Boa!
Morgado

Bruno wrote:

Apesar do que vou escrever não é uma prova, ajuda...

Se 1+1+1=3
posso dividir ambos os lados por 3:
0,. +0,333+0,333... = 1
Portanto: 0, =1

Até

=
Instruções para entrar na lista, sair da lista e usar a lista em
http://www.mat.puc-rio.br/~nicolau/olimp/obm-l.html
O administrador desta lista é [EMAIL PROTECTED]
=




=
Instruções para entrar na lista, sair da lista e usar a lista em
http://www.mat.puc-rio.br/~nicolau/olimp/obm-l.html
O administrador desta lista é [EMAIL PROTECTED]
=



Re: [obm-l] Re: [obm-l] questão IME

2002-08-10 Por tôpico Augusto César Morgado



a=b (le-se a eh congruo a b) (na realidade o sinal que se usa eh o de igual
com tres tracinhos) modulo p
significa
a-b eh multiplo de p
ou, o que eh o mesmo,
a e b deixam restos iguais na divisao por p.

Igor GomeZZ wrote:
[EMAIL PROTECTED]">
  Em 10/8/2002, 18:12, Eder ([EMAIL PROTECTED]) disse:
  
i)Pelo Pequeno Teorema de Fermat,temos que k^5=k (mod 5)

Pode parecer idiota, mas o que eh "mod 5"?Fui!### Igor GomeZZ  UIN: 29249895 Vitria, Esprito Santo, Brasil Criao: 10/8/2002 (22:48)Pare para pensar:Amigo: algum que sabe de tudo ateu respeito e gosta de ti assimmesmo. (Elbert Hubbard)=Instrues para entrar na lista, sair da lista e usar a lista emhttp://www.mat.puc-rio.br/~nicolau/olimp/obm-l.htmlO administrador desta lista  [EMAIL PROTECTED]
=






  1   2   3   >